Sie sind auf Seite 1von 82

THERMODYNAMICS II

Lecture Manual

________________________________________________

PART I

Basic Concepts of Thermodynamics


Properties of Pure Substances
First Law of Thermodynamics

Intended Learning Outcomes:


1. Ability to apply knowledge of mathematics and science to solve engineering
problems.
2. Familiar with the Basic Concepts of Thermodynamics and its Application.
3. Recognize the Properties of Pure Substances and First Law of Thermodynamics.

Contents
1. Basic Concepts of Thermodynamics
2. Properties of Pure Substances
3. First Law of Thermodynamics

1. BASIC CONCEPTS OF THERMODYNAMICS

1.1 Definition Of Thermodynamics


Thermodynamics may be defined as follows :
 Thermodynamics is an axiomatic science which deals with the relations
among heat, work and properties of system which are in equilibrium. It
describes state and changes in state of physical systems.
 Thermodynamics is the science of the regularities governing processes of
energy conversion.
 Thermodynamics is the science that deals with the interaction between
energy and material systems.
Thermodynamics, basically entails four laws or axioms known as Zeroth, First,
Second and Third law of thermodynamics.
 The First law throws light on concept of internal energy.
 The Zeroth law deals with thermal equilibrium and establishes a concept of
temperature.
 The Second law indicates the limit of converting heat into work and introduces the
principle of increase of entropy.
 The Third law defines the absolute zero of entropy.
These laws are based on experimental observations and have no mathematical
proof. Like all physical laws, these laws are based on logical reasoning.

1.2 Thermodynamic Systems


1.2.1 System, Boundary and Surroundings
System. A system is a finite quantity of matter or a prescribed region of
space (Refer Fig. 1)
Boundary. The actual or hypothetical envelope enclosing the system is the
boundary of the system. The boundary may be fixed or it may move, as and
when a system containing a gas is compressed or expanded. The boundary
may be real or imaginary. It is not difficult to envisage a real boundary but
an example of imaginary boundary would be one drawn around a system
consisting of the fresh mixture about to enter the cylinder of an I.C. engine
together with the remanants of the last cylinder charge after the exhaust
process (Refer Figure 2)

Figure 1 Figure 2

1.2.2 Types of thermodynamic systems:


1. Open system - a thermodynamic system for which there is exchange of
energy and matter with the surroundings
Figure 3
2. Closed system - a thermodynamic system for which there is only exchange
of energy with the surroundings, but the mass of the system remains
constant

Figure 4
3. Isolated System, which can be differentiated into
Thermally isolated system where there is no exchange of energy and
matter with the surroundings, and
Mechanically isolated system where there is no work done on the
system or by the system
4. Adiabatic system - is one which is thermally insulated from its
surroundings. It can, however, exchange work with its surroundings. If it
does not, it becomes an isolated system.
Phase. A phase is a quantity of matter which is homogeneous
throughout in chemical composition and physical structure.
5. Homogeneous System - A system which consists of a single phase is
termed as homogeneous system. Examples: Mixture of air and water
vapour, water plus nitric acid and octane plus heptane.
6. Heterogeneous System - A system which consists of two or more phases
is called a heterogeneous system. Examples: Water plus steam, ice plus
water and water plus oil.

1.3 Pure Substance


A pure substance is one that has a homogeneous and invariable chemical
composition even though there is a change of phase. In other words, it is a system
which is (a) homogeneous in composition, (b) homogeneous in chemical
aggregation. Examples : Liquid, water, mixture of liquid water and steam, mixture
of ice and water. The mixture of liquid air and gaseous air is not a pure substance.

1.4 Thermodynamic Equilibrium


A system is in thermodynamic equilibrium if the temperature and pressure
at all points are same ; there should be no velocity gradient ; the chemical
equilibrium is also necessary. Systems under temperature and pressure equilibrium
but not under chemical equilibrium are sometimes said to be in metastable
equilibrium conditions. It is only under thermodynamic equilibrium conditions that
the properties of a system can be fixed.
Thus for attaining a state of thermodynamic equilibrium the following three
types of equilibrium states must be achieved :
1. Thermal equilibrium. The temperature of the system does not change
with time and has same value at all points of the system.
2. Mechanical equilibrium. There are no unbalanced forces within the
system or between the surroundings. The pressure in the system is same at
all points and does not change with respect to time.
3. Chemical equilibrium. No chemical reaction takes place in the system
and the chemical composition which is same throughout the system does
not vary with time.

1.5 Properties Of Systems


A property of a system is a characteristic of the system which depends upon its
state, but not upon how the state is reached. There are two sorts of property :
1. Intensive properties. These properties do not depend on the mass of the system.
Examples : Temperature and pressure.
2. Extensive properties. These properties depend on the mass of the system.
Example : Volume. Extensive properties are often divided by mass associated with
them to obtain the intensive properties. For example, if the volume of a system of
mass m is V, then the specific volume of matter within the system is V/m = v which
is an intensive property.

1.6 State
State is the condition of the system at an instant of time as described or measured
by its properties. Or each unique condition of a system is called a state.
It follows from the definition of state that each property has a single value at each
state. Stated differently, all properties are state or point functions. Therefore, all properties
are identical for identical states.
On the basis of the above discussion, we can determine if a given variable is
property or not by applying the following tests:
- A variable is a property, if and only if, it has a single value at each equilibrium
state.
- A variable is a property, if and only if, the change in its value between any two
prescribed equilibrium states is single-valued.
Therefore, any variable whose change is fixed by the end states is a property.

1.7 Process
A process occurs when the system undergoes a change in a state or an energy
transfer at steady state. A process may be non-flow in which a fixed mass within the
defined boundary is undergoing a change of state. Closed systems undergo non-flow
processes. A process may be a flow process in which mass is entering and leaving through
the boundary of an open system.
In a steady flow process, mass is crossing the boundary from surroundings at entry,
and an equal mass is crossing the boundary at the exit so that the total mass of the system
remains constant.
Quasi-static process. Quasi means ‘almost’. A quasi-static process is also called
a reversible process. This process is a succession of equilibrium states and infinite
slowness is its characteristic feature.

1.8 Cycle
Any process or series of processes whose end states are identical is termed a cycle.
The processes through which the system has passed can be shown on a state diagram, but
a complete section of the path requires in addition a statement of the heat and work crossing
the boundary of the system

1.9 Point Function


When two properties locate a point on the graph (co-ordinate axes) then those
properties are called as point function.
Examples. Pressure, temperature, volume etc.

1.10 Path Function


There are certain quantities which cannot be located on a graph by a point but are
given by the area or so, on that graph. In that case, the area on the graph, pertaining to the
particular process, is a function of the path of the process. Such quantities are called path
functions.
Examples. Heat, work etc.
Heat and work are inexact differentials. Their change cannot be written as
difference between their end states.

1.11 Temperature
 The temperature is a thermal state of a body which distinguishes a hot body from
a cold body. The temperature of a body is proportional to the stored molecular
energy i.e., the average molecular kinetic energy of the molecules in a system. (A
particular molecule does not hhave a temperature, it has energy. The gas as a
system has temperature).

 Instruments for measuring ordinary temperatures are known as thermometers and


those for measuring high temperatures are known as pyrometers.
 It has been found that a gas will not occupy any volume at a certain temperature.
This temperature is known as absolute zero temperature. The temperatures
measured with absolute zero as basis are called absolute temperatures. Absolute
temperature is stated in degrees centigrade. The point of absolute temperature is
found to occur at 273.15°C below the freezing point of water.
Then : Absolute temperature = Thermometer reading in °C + 273.15.
Absolute temperature is degree centigrade is known as degrees kelvin, denoted by K
(SI unit).

1.12 Zeroth Law Of Thermodynamics


 ‘Zeroth law of thermodynamics’ states that if two systems are each equal in
temperature to a third, they are equal in temperature to each other.
 This law was enunciated by R.H. Fowler in the year 1931. However, since the first
and second laws already existed at that time, it was designated as zeroth law so that
it precedes the first and second laws to form a logical sequence.

1.13 Pressure
Pressure is defined as a force per unit area. Pressures are exerted by gases, vapours
and liquids. The instruments that we generally use, however, record pressure as the
difference between two pressures. Thus, it is the difference between the pressure exerted
by a fluid of interest and the ambient atmospheric pressure.
Such devices indicate the pressure either above or below that of the atmosphere.
When it is above the atmospheric pressure, it is termed gauge pressure and is positive.
When it is below atmospheric, it is negative and is known as vacuum. Vacuum readings
are given in millimetres of mercury or millimetres of water below the atmosphere.
It is necessary to establish an absolute pressure scale which is independent of the
changes in atmospheric pressure. A pressure of absolute zero can exist only in complete
vacuum.
Mathematically :
(i) Absolute pressure = Atmospheric pressure + Gauge pressure
pabs. = patm. + pgauge.
(ii) Vacuum pressure = Atmospheric pressure – Absolute pressure.
Vacuum is defined as the absence of pressure. A perfect vacuum is
obtained when absolute pressure is zero, at this instant molecular
momentum is zero.

1.14 Specific Volume


The specific volume of a system is the volume occupied by the unit mass of the
system. The symbol used is v and units are ; for example, m3/kg. The symbol V will be
used for volume. (Note that specific volume is reciprocal of density).
1.15 Reversible And Irreversible Processes
Reversible process. A reversible process (also sometimes known as quasi-static
process) is one which can be stopped at any stage and reversed so that the system and
surroundings are exactly restored to their initial states.
This process has the following characteristics:
1. It must pass through the same states on the reversed path as were initially
visited on the forward path.
2. This process when undone will leave no history of events in the
surroundings.
3. It must pass through a continuous series of equilibrium states.
No real process is truely reversible but some processes may approach reversibility,
to close approximation.

Examples. Some examples of nearly reversible processes are:


(i) Frictionless relative motion.
(ii) Expansion and compression of spring.
(iii) Frictionless adiabatic expansion or compression of fluid.
(iv) Polytropic expansion or compression of fluid.
(v) Isothermal expansion or compression.
(vi) Electrolysis.
Irreversible process. An irreversible process is one in which heat is transferred
through a finite temperature.
Examples.
(i) Relative motion with friction
(ii) Combustion
(iii) Diffusion
(iv) Free expansion
(v) Throttling
(vi) Electricity flow through a resistance
(vii) Heat transfer
(viii) Plastic deformation.
An irreversible process is usually represented by a dotted (or discontinuous) line
joining the end states to indicate that the intermediate states are indeterminate.
Irreversibilities are of two types:
1. External irreversibilities. These are associated with dissipating effects outside
the working fluid.
Example. Mechanical friction occurring during a process due to some external
source.
2. Internal irreversibilities. These are associated with dissipating effects within
the working fluid.
Example. Unrestricted expansion of gas, viscosity and inertia of the gas.

1.16 Energy, Work And Heat


1.16.1. Energy
Energy is a general term embracing energy in transition and stored energy.
The stored energy of a substance may be in the forms of mechanical energy and
internal energy (other forms of stored energy may be chemical energy and electrical
energy). Part of the stored energy may take the form of either potential energy
(which is the gravitational energy due to height above a chosen datum line) or
kinetic energy due to velocity. The balance part of the energy is known as internal
energy.
In a non-flow process usually there is no change of potential or kinetic
energy and hence change of mechanical energy will not enter the calculations. In
a flow process, however, there may be changes in both potential and kinetic energy
and these must be taken into account while considering the changes of stored
energy. Heat and work are the forms of energy in transition. These are the only
forms in which energy can cross the boundaries of a system. Neither heat nor work
can exist as stored energy.

1.16.2. Work and Heat


Work is said to be done when a force moves through a distance. If a part
of the boundary of a system undergoes a displacement under the action of a
pressure, the work done W is the product of the force (pressure × area), and the
distance it moves in the direction of the force.
Sign convention :
 If the work is done by the system on the surroundings, e.g., when a fluid
expands pushing a piston outwards, the work is said to be positive.
i.e., Work output of the system = + W
 If the work is done on the system by the surroundings, e.g., when a force is
applied to a rotating handle, or to a piston to compress a fluid, the work is
said to be negative.
i.e., Work input to system = – W

Heat (denoted by the symbol Q), may be, defined in an analogous way to
work as follows :
“Heat is ‘something’ which appears at the boundary when a system
changes its state due to a difference in temperature between the system and its
surroundings”.
Heat, like work, is a transient quantity which only appears at the boundary
while a change is taking place within the system.
It is apparent that neither δW or δQ are exact differentials and therefore any
integration of the elemental quantities of work or heat which appear during a change
from state 1 to state 2 must be written as

Sign convention :
If the heat flows into a system from the surroundings, the quantity is said to
be positive and, conversely, if heat flows from the system to the surroundings it is
said to be negative.
In other words :
Heat received by the system = + Q
Heat rejected or given up by the system = – Q.

Comparison of Work and Heat


Similarities :
(i) Both are path functions and inexact differentials.
(ii) Both are boundary phenomenon i.e., both are recognized at the boundaries of
the system as they cross them.
(iii) Both are associated with a process, not a state. Unlike properties, work or heat
has no meaning at a state.
(iv) Systems possess energy, but not work or heat.

Dissimilarities :
(i) In heat transfer temperature difference is required.
(ii) In a stable system there cannot be work transfer, however, there is no
restriction for the transfer of heat.
(iii) The sole effect external to the system could be reduced to rise of a weight but
in the case of a heat transfer other effects are also observed.
SAMPLE PROBLEMS:
Basic Concepts of Thermodynamics
Example 1:
A tube contains an oil of specific gravity 0.9 to a depth of 120 cm. Find the gauge
pressure at this depth (in kN/m2).

Example 2:
An artificial satellite revolves round the earth with a relative velocity of 800 m/s. If
acceleration due to gravity is 9 m/s2 and gravitational force is 3600 N, calculate its
kinetic energy.

Example 3:
A vacuum recorded in the condenser of a steam power plant is 740 mm of Hg. Find the
absolute pressure in the condenser in Pa. The barometric reading is 760 mm of Hg.
Example 4:
The specific heat capacity of the system during a certain process is given by
cn = (0.4 + 0.004 T) kJ/kg°C.
If the mass of the gas is 6 kg and its temperature changes from 25°C to 125°C find :
(i) Heat transferred
(ii) Mean specific heat of the gas.

Example 5:
The properties of a closed system change following the relation between pressure and
volume as pV = 3.0 where p is in bar V is in m3. Calculate the work done when the
pressure increases from 1.5 bar to 7.5 bar.
2. PROPERTIES OF PURE SUBSTANCES

2.1 Definition of Pure Substance

A pure substance is a system which is (i) homogeneous in composition, (ii)


homogeneous in chemical aggregation, and (iii) invariable in chemical
aggregation.
 “Homogeneous in composition” means that the composition of each part of
the system is the same as the composition of every other part. “Composition
means the relative proportions of the chemical elements into which the
sample can be analysed. It does not matter how these elements are
combined.
 “Homogeneous in chemical aggregation” means that the chemical
elements must be combined chemically in the same way in all parts of the
system.
 “Invariable in chemical aggregation” means that the state of chemical
combination of the system does not change with time (condition (ii) referred
to variation with position). Thus a mixture of hydrogen and oxygen, which
changed into steam during the time that the system was under consideration,
would not be a pure substance.

2.2 Phase Change of Pure Substance

Figure 3 Figure 4
Vapour pressure curve - For a pure substance, definite relationship exists between
the saturation pressure and saturation temperature
Subcooled liquid - if the temperature of the liquid water on cooling becomes lower
than the saturation temperature for the given pressure, the liquid water
Compressed liquid - the pressure on the liquid water is greater than the saturation
pressure at a given temperature. In this condition, the liquid water
The term compressed liquid or sub-cooled liquid is used to distinguish it
from saturated liquid. All points in the liquid region indicate the states of the
compressed liquid.
Superheated vapour - When all the liquid has been evaporated completely and heat
is further added, the temperature of the vapour increases.
The difference between the superheated temperature and the saturation
temperature at the given pressure is called the degree of superheat.
From the heating process at a constant pressure of 225 bar represented by
the curve 9-10-11 in Fig.3, it can be seen that there is no constant temperature
vapourisation line. The specific volume of the saturated liquid and of the saturated
vapour is the same, i.e., vf = vg. Such a state of the substance is called the critical
state. The parameters like temperature, pressure, volume, etc. at such a state are
called critical parameters.
The curve 12-13 (Fig.3) represents a constant pressure heating process,
when the pressure is greater than the critical pressure. At this state, the liquid water
is directly converted into superheated steam. As there is no definite point at which
the liquid water changes into superheated steam, it is generally called liquid water
when the temperature is less than the critical temperature and superheated steam
when the temperature is above the critical temperature.

2.3 p-T (Pressure-Temperature) Diagram For A Pure Substance


The points representing the
coexistence of
(i) solid and vapour lie on the
‘sublimation curve’,
(ii) liquid and vapour lie on the
‘vapourisation curve’,
(iii) liquid and solid lie on the
‘fusion curve’. In the
particular case of water,
the sublimation curve is called
the frost line, the
vapourisation curve is called
the steam line, and the Figure 5
fusion curve is called the ice line.

The slopes of sublimation and the vapourisation curves for all substances are
positive. The slope of the fusion curve, however may be positive or negative. The fusion
curve of most substances have a positive slope. Water is one of the important exceptions.

Triple point
The triple point is merely the point of intersection of sublimation and
vapourisation curves. It must be understood that only on p-T diagram is the triple
point represented by a point. On p-V diagram it is a line, and on a U-V diagram it
is a triangle.
The pressure and temperature at which all three phases of a pure substance
coexist may be measured with the apparatus that is used to measure vapour
pressure.

2.4 Phase Change Terminology And Definitions Suffices :

Suffices: Solid i
Liquid f
Vapour g

Phase Change Name Process Process Suffix


1. Solid-Liquid Fusion Freezing, Melting if
2. Solid-Vapour Sublimation Frosting, Defrosting ig
3. Liquid-Vapour Evaporation Evaporating, Condensing fg

Critical pressure. The pressure at the critical point.


Critical temperature. The temperature at the critical point.
Gas. A vapour whose temperature is greater than the critical temperature.
Liquid-vapour terms :
Saturation temperature. The phase change
temperature corresponding to the saturation
pressure. Sometimes called the boiling temperature.
Saturation pressure. The phase change pressure.

Compressed liquid. Liquid whose temperature is


lower than the saturation temperature. Sometimes
called a sub-cooled liquid.

Saturated liquid. Liquid at the saturation


temperature corresponding to the saturation
pressure. That is liquid about to commence evaporating, represented by the point f on a
diagram.

Saturated vapour. A term including wet and dry vapour.

Dry (saturated) vapour. Vapour which has just completed evaporation. The pressure and
temperature of the vapour are the saturation values. Dry vapour is represented by a point g
on a diagram.

Wet vapour. The mixture of saturated liquid and dry vapour during the phase change.
Superheated vapour. Vapour whose temperature is greater than the saturation temperature
corresponding to the pressure of the vapour.

Degree of superheat. The term used for the numerical amount by which the temperature
of a superheated vapour exceeds the saturation temperature.

2.5 Important Terms Relating Steam Formation


1. Sensible heat of water (hf ).
It is defined as the quantity of heat absorbed by 1 kg of water when it is heated from
0°C (freezing point) to boiling point. It is also called total heat (or enthalpy) of water or
liquid heat invariably. It is reckoned from 0°C where sensible heat is taken as zero.
Note. The value of specific heat of water may be taken as 4.18 kJ/kg K at low pressures but at high
pressures it is different from this value.

2. Latent heat or hidden heat (hfg).


It is the amount of heat required to convert water at a given temperature and
pressure into steam at the same temperature and pressure. It is expressed by the symbol
hfg and its value is available from steam tables. The value of latent heat is not constant and
varies according to pressure variation.

3. Dryness fraction (x).


The term dryness fraction is related with wet steam. It is defined as the ratio of the
mass of actual dry steam to the mass of steam containing it. It is usually expressed by the
symbol ‘x’ or ‘q’.
If ms = Mass of dry steam contained in steam considered, and
mw = Weight of water particles in suspension in the steam considered,
Then,

Thus if in 1 kg of wet steam 0.9 kg is the dry steam and 0.1 kg water particles then
x = 0.9. Note. No steam can be completely dry and saturated, so long as it is in contact with the
water from which it is being formed.

4. Total heat or enthalpy of wet steam (h).


It is defined as the quantity of heat required to convert 1 kg of water at 0°C into
wet steam at constant pressure. It is the sum of total heat of water and the latent heat and
this sum is also called enthalpy.
In other words, h = hf + xhfg
If steam is dry and saturated, then x = 1 and hg = hf + hfg.

5. Superheated steam.
When steam is heated after it has become dry and saturated, it is called superheated
steam and the process of heating is called superheating. Superheating is always carried
out at constant pressure. The additional amount of heat supplied to the steam during
superheating is called as ‘Heat of superheat’ and can be calculated by using the specific
heat of superheated steam at constant pressure (cps), the value of which varies from 2.0 to
2.1 kJ/ kg K depending upon pressure and temperature

6. Volume of wet and dry steam.


If the steam has dryness fraction of x, then 1 kg of this steam will contain x kg of
dry steam and (1 – x) kg of water. If vf is the volume of 1 kg of water and vg is the volume
of 1 kg of perfect dry steam (also known as specific volume), then volume of 1 kg of wet
steam = volume of dry steam + volume of water. = xvg + (1 – x)vf ..
7. Volume of superheated steam.
As superheated steam behaves like a perfect gas its volume can be found out in the
same way as the gases.

2.6 Internal Energy Of Steam


It is defined as the actual energy stored in the steam. As per previous articles, the
total heat of steam is sum of sensible heat, internal latent heat and the external work of
evaporation. Work of evaporation is not stored in the steam as it is utilised in doing external
work. Hence the internal energy of steam could be found by subtracting work of
evaporation from the total heat.

2.7 Determination of Dryness Fraction of Steam


The dryness fraction of steam can be measured by using the following calorimeters
1. Tank or bucket calorimeter
A known mass of steam is passed through a known mass of water
and steam is completely condensed. The heat lost by steam is equated to
heat gained by the water.

2. Throttling calorimeter
The steam to be sampled is taken from the pipe by means of suitable
positioned and dimensioned sampling tube. It passes into an insulated
container and is throttled through an orifice to atmospheric pressure.
3. Separating and throttling calorimeter
If the steam whose dryness fraction is to be determined is very wet
then throttling to atmospheric pressure may not be sufficient to ensure
superheated steam at exit. In this case it is necessary to dry the steam
partially, before throttling.

SAMPLE PROBLEMS:
Properties of Pure Substances
Example 1:

Calculate the dryness fraction (quality) of steam which has 1.5 kg of water in suspension
with 50 kg of steam.

Example 2:
A vessel having a volume of 0.6 m3 contains 3.0 kg of liquid water and water vapour
mixture in equilibrium at a pressure of 0.5 MPa. Calculate :
(i) Mass and volume of liquid ;

ii. Mass and volume of vapour.

Example 3:
What amount of heat would be required to produce 4.4 kg of steam at a pressure of 6 bar
and temperature of 250°C from water at 30°C ? Take specific heat for superheated steam
as 2.2 kJ/kg K.

Example 4:
Determine the mass of 0.15 m3 of wet steam at a pressure of 4 bar and dryness fraction
0.8. Also calculate the heat of 1 m3 of steam.

Example 5:
Find the specific volume, enthalpy and internal energy of wet steam at 18 bar, dryness
fraction 0.85
3 FIRST LAW OF THERMODYNAMICS

3.1 Internal Energy

It is the heat energy stored in a gas. If a certain amount of heat is supplied


to a gas the result is that temperature of gas may increase or volume of gas may
increase thereby doing some external work or both temperature and volume may
increase ; but it will be decided by the conditions under which the gas is supplied
heat. If during heating of the gas the temperature increases its internal energy will
also increase.
Joule’s law of internal energy states that internal energy of a perfect gas is
a function of temperature only. In other words, internal energy of a gas is dependent
on the temperature change only and is not affected by the change in pressure and
volume.

3.2 Law of Conservation of Energy

In the early part of nineteenth century the scientists developed the concept
of energy and hypothesis that it can be neither created nor destroyed ; this came to
be known as the law of the conservation of energy. The first law of thermodynamics
is merely one statement of this general law/principle with particular reference to
heat energy and mechanical energy i.e., work.

3.3 First Law of Thermodynamics


“When a system undergoes a thermodynamic cycle then the net heat
supplied to the system from the surroundings is equal to net work done by the
system on its surroundings.

The First Law of Thermodynamics may also be stated as follows :


“Heat and work are mutually convertible but since energy can neither
be created nor destroyed, the total energy associated with an energy conversion
remains constant”.
“No machine can produce energy without corresponding expenditure
of energy, i.e., it is impossible to construct a perpetual motion machine of first
kind”.

3.4 Application of First Law to Process

When a process is executed by a system, the change in stored energy of the


system is numerically equal to the net heat interactions minus the net work
interaction during the process.

If the electric, magnetic and chemical energies are absent and changes in
potential and kinetic energy for a closed system are neglected, the above equation
can be written as

Generally, when heat is added to a system its temperature rises and external
work is performed due to increase in volume of the system. The rise in temperature
is an indication of increase of internal energy. Heat added to the system will be
considered as positive and the heat removed or rejected, from the system, as
negative.

3.5 Perpetual Motion Machine of the First Kind- PMM1

The first law of thermodynamics states the general principle of the


conservation of energy. Energy is neither created nor destroyed, but only gets
transformed from one form to another. There can be no machine which would
continuously supply mechanical work without some form of energy disappearing
simultaneously. Such a fictitious machine is called a perpetual motion machine of
the first kind, or in brief, PMM 1. A PMM 1 is thus impossible.

3.6 Energy of an Isolated System


An isolated system is one in which there is no interaction of the system with
the surroundings. For an isolated system,

3.7 Specific Heats

The specific heat of a solid or liquid is usually defined as the heat required
to raise unit mass through one degree temperature rise.

For small quantities, we have

For a gas there are an infinite number of ways in which heat may be added
between any two temperatures, and hence a gas could have an infinite number of
specific heats. However, only two specific heats for gases are defined.

Specific heat at constant volume, cv and,


Specific heat at constant pressure, cp.
We have
dQ = m cp dT For a reversible non-flow process at constant pressure and,
dQ = m cv dT For a reversible non-flow process at constant volume

Flow of heat in a reversible constant pressure process = mcp (T2 – T1)


Flow of heat in a reversible constant volume process = mcv (T2 – T1)
In case of real gases, cp and cv vary with temperature, but a suitable average
value may be used for most practical purposes.

3.8 Joules Law

Joule’s law states as follows :


“The internal energy of a perfect gas is a function of the absolute
temperature only.”
i.e., u = f(T)
To evaluate this function let 1 kg of a perfect gas be heated at
constant volume.
According to non-flow energy equation,
dQ = du + dW
dW = 0, since volume remains constant
∴ dQ = du

At constant volume for a perfect gas, for 1 kg


dQ = cvdT
∴ dQ = du = cvdT
and integrating u = cv T + K, K being constant.

According to Joule’s law u = f(T), which means that internal energy varies
linearly with absolute temperature. Internal energy can be made zero at any
arbitrary reference temperature. For a perfect gas it can be assumed that u = 0 when
T = 0, hence constant K is zero.
i.e., Internal energy, u = cv T for a perfect gas
or For mass m, of a perfect gas
Internal energy, U = mcv T
For a perfect gas, in any process between states 1 and 2, we have from Eqn. (4.21)
Gain in internal energy,
U2 – U1 = mcv (T2 – T1)

3.9 Enthalpy

One of the fundamental quantities which occur invariably in


thermodynamics is the sum of internal energy (u) and pressure volume product (pv).
This sum is called Enthalpy (h).
i.e., h = u + pv ...(4.24)
The enthalpy of a fluid is the property of the fluid, since it consists of the
sum of a property and the product of the two properties. Since enthalpy is a property
like internal energy, pressure, specific volume and temperature, it can be introduced
into any problem whether the process is a flow or a non-flow process.
The total enthalpy of mass, m, of a fluid can be
H = U + pV, where H = mh.

3.10 Application of First Law of Thermodynamics to Non-Flow or Closed


System

Table 1 Summary of Processes for Perfect Gas (Unit mass)


SAMPLE PROBLEMS:
First Law of Thermodynamics
Example 1:
In an internal combustion engine, during the compression stroke the heat rejected to the
cooling water is 50 kJ/kg and the work input is 100 kJ/kg. Calculate the change in internal
energy of the working fluid stating whether it is a gain or loss.

Example 2:
In an air motor cylinder the compressed air has an internal energy of 450 kJ/kg at the
beginning of the expansion and an internal energy of 220 kJ/kg after expansion. If the work
done by the air during the expansion is 120 kJ/kg, calculate the heat flow to and from the
cylinder.
Example 3:
0.3 kg of nitrogen gas at 100 kPa and 40°C is contained in a cylinder. The piston is moved
compressing nitrogen until the pressure becomes 1 MPa and temperature becomes 160°C.
The work done during the process is 30 kJ. Calculate the heat transferred from the nitrogen
to the surroundings. cv for nitrogen = 0.75 kJ/kg K.

Example 4:
When a stationary mass of gas was compressed without friction at constant pressure its
initial state of 0.4 m3 and 0.105 MPa was found to change to final state of 0.20 m3 and
0.105 MPa. There was a transfer of 42.5 kJ of heat from the gas during the process. How
much did the internal energy of the gas change ?
Example 5:
A container is divided into compartments by a partition. The container is completely
insulated so that there is no heat transfer. One portion contains gas at temperature T1 and
pressure p1 while the other portion also has the same gas but at temperature T2 and pressure
p2. How will the First Law of Thermodynamics conclude the result if partition is removed
?
REFERENCES

Books

[1] Smith, J., Van Ness, H., & Abbott, M. (2005). Introduction to chemical engineering
thermodynamics. Boston, Mass.: McGraw-Hill.

[2] Rajput, R. (2011). Engineering thermodynamics. New Delhi: Firewal media.

[3] Borgnakke, C., Sonntag, R., Van Wylen, G., & Sonntag, R. (2009). Fundamentals
of thermodynamics. Hoboken, NJ: Wiley.
THERMODYNAMICS II
Lecture Manual

________________________________________________

PART II

Second Law of Thermodynamics


Heat Engines
Entropy and Entropy Balances
Third Law of Thermodynamics

Intended Learning Outcomes


1. Instruct models, principles and calculations for better understanding on the Second
and Third Law of Thermodynamics and its applications.
2. Convey Carnot ideologies, reversed Carnot and correlate to the Second Law.
3. Present the principles and calculations of Entropy and Entropy Balances.
4. Exercise analysis and answering problems pertaining to Second and Third Law of
Thermodynamics, Carnot principle, Entropy and Entropy Balances.
5. Appreciate the portents after explicit conditions and applications of the topic.
THE SECOND LAW OF THERMODYNAMICS

Introduction
Energy’s value cannot be conserved and some energy must be perpetually reduced
to a lesser value whenever energy transfer occurs. “Whenever energy is transferred, energy
must be conserved, but the value of energy cannot be all consumed and some energy must
be permanently reduced to a lower value.”
Second Law offers a way of foreseeing the course of processes; forming
equilibrium conditions; defining the greatest theoretical cycle performance and engine;
assessing quantitatively the factors that affect the greatest theoretical cycle performance
and engine; outlining a temperature scale autonomous of the thermometric properties of
any substance; and lastly, developing means for evaluating U and H in terms of
thermodynamic properties which can be experimentally obtained.

Statements of the Second Law of Thermodynamics


1. The Principle of Thomson (Lord Kelvin) - “It is impossible by a cyclic process to
take heat from a reservoir and to convert it into work without simultaneously
transferring heat from a hot to a cold reservoir.”
2. The Principle of Clausius -”It is impossible to devise an engine which, working in
a cycle, shall produce no effect other than the transfer of heat from a colder to a
hotter body.”

The left diagram shows a refrigerator (R) taking heat, Q, from a low-
temperature reservoir and transferring it to a high-temperature reservoir. The
Second Law says, "NO!" to such a device.
The center and right diagrams show what is possible. The center diagram
shows heat being transferred from a high-temperature reservoir to a device that
transfers it to a low-temperature reservoir. This is the opposite of what is forbidden.
We know that heat will naturally flow from high temperature to low temperature,
even without any device in between. Actual refrigerators and heat pumps operate
by putting work, W, into the device (R), taking heat, QL, from the low-temperature
reservoir and transferring a different amount of heat, QH, to the high-temperature
reservoir, as shown by the right diagram.
3. The Principle of Planck - “It is impossible to devise an engine, which working in a
complete cycle will produce no effect other than raising of a weight and the cooling
of a heat reservoir.”

The left diagram shows an engine (E) taking heat from a reservoir and
converting it to work. According to the First Law, W = Q. The Second Law says
"NO!" to this. What is possible is shown in the center and right diagrams. The center
diagram shows work being converted completely to heat, exactly the opposite of
what is forbidden. This is readily done, because of friction. Think of what happens
when you drive your car with the brakes on. The work of the engine is converted to
heat in the brakes, which is eventually transferred to the surrounding atmosphere.
The right diagram shows a typical engine that can be made to work. Heat, QH, is
transferred to the engine from a reservoir at a high temperature. Part of this energy
is converted to work, W, which is delivered to an external load, and part of the
energy, QL, is transferred as heat to a low temperature reservoir.

Importance of Second Law of Thermodynamics


 It forms an energy quality level
 It defines the flow direction
 It creates ideal limit when a device yields or expenditures work in a cyclic process
 It predetermines how a system will change

Perpetual Motion Machine


A perpetual machine would have no friction and thus would run forever but yields
no work. It generates work from nothing or generates mass or energy which violates the 1st
law. It harvests heat from a source and then change this heat entirely into other energy
forms which violates the 2nd law.
1. First Kind - In imaginary device which would produce work continuously without
absorbing any energy from its surroundings is called a Perpetual Motion Machine
of the First kind, (PMMFK). A PMMFK is a device which violates the first law of
thermodynamics. It is impossible to devise a PMMFK.
The converse of the above statement is also true, i.e., there can be no
machine which would continuously consume work without some other form of
energy appearing simultaneously.

2. Second Kind - A machine that exchanges heat with a single thermal reservoir, while
providing work that equals the amount of heat exchanged with the reservoir, is
called Perpetual Motion Machine of Second Kind.

It violates the Second Law of Thermodynamics, both the Kelvin-Planck and the
Clausius statements.

HEAT ENGINES

Are device of machines that produces work from heat in a cyclic process. Closed system
that exchanges only heat and work with its surrounding and that operates in cycles.
Example of which is a combustion engine and turbine engine or steam power plant.

Elements of Heat Engine


1. Working substance: matter, fluid that receives heat, rejects heat and does work.
2. Source of Heat (hot body, heat reservoir, source): from which the working
substance receives heat
3. Heat Sink (receiver, a cold body, sink): to which he working substance can reject
heat
4. Engine: wherein the working substance mat do work or have work done on it.

The Carnot Cycle

The Carnot heat engine is a hypothetical, ideal engine that operates on the reversible
Carnot cycle. It is used as a reference cycle although ironically, no real Carnot Engines
are known to have been made. It is a closed cycle using the external application of heat.

The Carnot cycle when acting as a heat engine consists of the following steps:

Change
Carnot Heat Cycle Processes
of State
Reversible isothermal compression of the cold gas. Isothermal heat
A to B rejection. Gas starts at its "cold" temperature. Heat flows out of the gas to
the low temperature environment.
Reversible adiabatic compression of the gas. Compression causes the gas
B to C
temperature to rise to its "hot" temperature. No heat gained or lost.
Reversible isothermal expansion of the hot gas. Isothermal heat
C to D addition. Absorption of heat from the high temperature source. Expanding
gas available to do work on the surroundings (e.g. moving a piston).
Reversible adiabatic expansion of the gas. The gas continues to expand,
D to A doing external work. The gas expansion causes it to cool to its "cold"
temperature. No heat is gained or lost.

1. Isothermal Compression (A-B)


-dQ = PdV
-dQ = RTC dV/V
QC = RTC ln (VA/VB)

2. Reversible Adiabatic Compression (B-C)


∆U = Q + W
PdV = -nCv dT
-Cv dT = dV
R T V
(Cv/R) ln (Tc/TH) = ln (VB/VC)

3. Isothermal Expansion (C-D)


dQH = Cv dT + PdV
dQH = PdV
but, PV = RT
QH = RTH ln (VD/VC)

4. Reversible Adiabatic Expansion (D-A)


-dQ = Cv dT + PdV
-CvdT = PdV
-Cv/R dT/T = dV/V
(Cv/R) ln (TC/TH) = ln (VA/VD)

Equation derived from the above correlations


Combining equations (1) to (3):
(QH / QC) = (TH/TC)
Recalling engine efficiency equation:
η = W/QH = (QH – QC) / QH
= 1 – (QC/QH)
= 1 – (TC/TH)

Carnot Principles
 The thermal efficiency of an internally irreversible heat engine is always less than
the thermal efficiency of an internally reversible heat engine operating with heat
transfer at the same high- and low-temperature regions.
 The thermal efficiencies of two internally reversible heat engines operating with
heat transfer at the same high- and low-temperature regions are equal.

Carnot’s Engine Efficiency


Efficiency of a Carnot engine approach unity only when TH approaches infinity or
TC approaches zero. All heat engines operate at η less than unity. Two reversible devices
working at the same temperature limits will have the same η.
ηactual = ηcarnot (reversible process)
ηactual < ηcarnot (irreversible process)
ηactual > ηcarnot (violates 2nd law)

Sample Problem
An ideal gas heat engine operates in Carnot cycle between 227°C and 127°C.It absorbs
6x102 cal of heat at the higher temperature. Calculate the amount of heat supplied to the
engine from the source in each cycle.

Solution
T1 = 227°C = 500 K
T2= 127°C = 400 K
ηcarnot = 1 - (T2/T1) = 1/5
Now, also Efficiency = Heat supplied from source/Heat absorbed at high temperature
so Heat supplied from source = (6x102)(1/5) = 1.2x102 cal

Reversed Carnot engine


When mechanical work is supplied as an energy input and heat may be moved from
one energy value (temperature) to another energy value. Carnot heat pump operates in a
reverse heat engine. It transfers heat from cold to warm, if work is done on engine. COP is
the heat supplied to the cycle at a low temperature (the desired effect) divided by the net
work.

Heat pumps, air conditioners, and refrigerators are heat engines operated backward.
The one shown here is based on a Carnot (reversible) engine. (a) Schematic diagram
showing heat transfer from a cold reservoir to a warm reservoir with a heat pump. The
directions of W, QH, and QC are opposite what they would be in a heat engine. (b) PV-
diagram for a Carnot cycle similar to that in the figure but reversed, following path
ADCBA. The area inside the loop is negative, meaning there is a net work input. There is
heat transfer Qc into the system from a cold reservoir along path DC, and heat transfer QH
out of the system into a hot reservoir along path BA.

Coefficient of Performance (COP)


COP = QC / W
since: W = QH – QC or
W + QC = QH
then: COP= QC/(QH-QC)
= (QC/QH) – 1
= (TC/TH) – 1
= TC/(TH-TC)
also: QC/(QH-QC) = TC/(TH-TC)
Based on reversed Carnot engine (refrigeration)
COPC = TC/(TH-TC)
Based on Carnot engine (heat generation)
COPH = TH/(TH-TC)

Sample Problem
A heat pump used to warm a home must employ a cycle that produces a working fluid at
temperatures greater than typical indoor temperature so that heat transfer to the inside can
take place. Similarly, it must produce a working fluid at temperatures that are colder than
the outdoor temperature so that heat transfer occurs from outside. Its hot and cold reservoir
temperatures therefore cannot be too close, placing a limit on its COPhp. Is the best
coefficient of performance possible for such a heat pump, if it has a hot reservoir
temperature of 45.0°C and a cold reservoir temperature of −15.0°C?

Solution
Carnot efficiency in terms of absolute temperature is given by:
η = 1 − Tc/TH
The temperatures in Kelvin are TH = 318 K and Tc = 258 K, so that
η = 1 – 258 K/318 K = 0.1887
Thus, from the discussion above,
COPhp = 1/0.1887 = 5.30
or
COPhp = QH/W = 5.30
so that
QH = 5.30 W
This result means that the heat transfer by the heat pump is 5.30 times as much as the work
put into it. It would cost 5.30 times as much for the same heat transfer by an electric room
heater as it does for that produced by this heat pump. This is not a violation of conservation
of energy. Cold ambient air provides 4.3 J per 1 J of work from the electrical outlet.

ENTROPY
The second law leads to the definition of a new property called entropy.

The Clausius Inequality


The first law is simply an energy balance. However, the second law leads to an inequality;
an irreversible process is less efficient than a reversible process. Another important
inequality in thermodynamics is the Clausius inequality:
𝛿𝑄
∮ ≤0
𝑇
That is, the cyclic integral of δQ / T is always less than or equal to zero. This is valid for
all cycles, reversible or irreversible.
For internally reversible cycles, it can be shown that:
𝛿𝑄
∮ | ≤0
𝑇 𝑖𝑛𝑡,𝑟𝑒𝑣

Entropy
The Clausius inequality forms the basis for the definition of a new property called entropy.
As can be seen in the equation above, for an internally reversible process the cyclic
integral of δQ / T is zero. A quantity whose cyclic integral is zero depends on the state only
and not the process path, and thus it is a property.
Clausius in 1865 realized that he discovered a new property and he called it entropy:
𝛿𝑄
𝑑𝑆 = ( ) (𝑘𝐽/𝐾)
𝑇 𝑖𝑛𝑡,𝑟𝑒𝑣
Entropy per unit mass is designated by s (kJ/kg.K).
The entropy change of a system during a process can be calculated:

2
𝛿𝑄
∆𝑆 = 𝑆2 − 𝑆1 = ∫ ( ) (𝑘𝐽/𝐾)
1 𝑇 𝑖𝑛𝑡,𝑟𝑒𝑣
To perform this integral, one needs to know the relation between Q and T during
the process.
Note that the cyclic integral of δQ / T will give us the entropy change only if the
integration carried out along an internally reversible path between two states.
For irreversible processes, we may imagine a reversible process between the two states
(initial and final) and calculate the entropy change (since entropy is a property).

The Increase of Entropy Principle


Entropy change of a closed system during an irreversible process is greater that the
integral of δQ / T evaluated for the process. In the limiting case of a reversible process,
they become equal.
𝛿𝑄
𝑑𝑆 ≥
𝑇
The entropy generated during a process is called entropy generation, and is denoted by
Sgen,
2
𝛿𝑄
∆𝑆 = 𝑆2 − 𝑆1 = ∫ + 𝑆𝑔𝑒𝑛
1 𝑇

Note that the entropy generation Sgen is always a positive quantity or zero (reversible
process). Its value depends on the process, thus it is not a property of a system.
The entropy of an isolated system during a process always increases, or in the
limiting case of a reversible process remains constant (it never decreases). This is known
as the increase of entropy principle.
The entropy change of a system or its surroundings can be negative; but entropy
generation cannot.
>0 𝑖𝑟𝑟𝑒𝑣𝑒𝑟𝑠𝑖𝑏𝑏𝑙𝑒 𝑝𝑟𝑜𝑐𝑒𝑠𝑠
𝑆𝑔𝑒𝑛 = { = 0 𝑟𝑒𝑣𝑒𝑟𝑠𝑖𝑏𝑙𝑒 𝑝𝑟𝑜𝑐𝑒𝑠𝑠
<0 𝑖𝑚𝑝𝑜𝑠𝑠𝑖𝑏𝑙𝑒 𝑝𝑟𝑜𝑐𝑒𝑠𝑠

1‐ A process must proceeds in the direction that complies with the increase of entropy
principle, Sgen > 0. A process that violates this principle is impossible.
2‐ Entropy is a non‐conserved property, and there is no such thing as the conservation of
entropy. Therefore, the entropy of universe is continuously increasing.
3‐ The performance of engineering systems is degraded by the presence of irreversibility.
The entropy generation is a measure of the magnitudes of the irreversibility present
during the process.
Entropy Balance
Entropy is a measure of molecular disorder or randomness of a system, and the second law
states that entropy can be created but it cannot be destroyed.
The increase of entropy principle is expressed as

Entropy change = Entropy transfer + Entropy generation


∆𝑆𝑠𝑦𝑠𝑡𝑒𝑚 = 𝑆𝑡𝑟𝑎𝑛𝑠𝑓𝑒𝑟 + 𝑆𝑔𝑒𝑛
This is called the entropy balance.
Entropy Change
The entropy balance is easier to apply that energy balance, since unlike energy (which has
many forms such as heat and work) entropy has only one form. The entropy change for a
system during a process is:

Entropy change = Entropy at final state ‐ Entropy at initial state


∆𝑆𝑠𝑦𝑠𝑡𝑒𝑚 = 𝑆𝑓𝑖𝑛𝑎𝑙 − 𝑆𝑖𝑛𝑖𝑡𝑖𝑎𝑙

Therefore, the entropy change of a system is zero if the state of the system does
not change during the process. For example entropy change of steady flow devices such
as nozzles, compressors, turbines, pumps, and heat exchangers is zero during steady
operation.

Mechanisms of Entropy Transfer


Entropy can be transferred to or from a system in two forms: heat transfer and mass flow.
Thus, the entropy transfer for an adiabatic closed system is zero.

Heat Transfer: heat is a form of disorganized energy and some disorganization (entropy)
will flow with heat. Heat rejection is the only way that the entropy of a fixed mass can be
decreased. The ratio of the heat transfer Q/ T (absolute temperature) at a location
is called entropy flow or entropy transfer
𝑄
Entropy transfer with heat (T = constant) 𝑆ℎ𝑒𝑎𝑡 = 𝑇
Since T (in Kelvin) is always positive, the direction of entropy transfer is the same of the
direction of heat transfer.
When two systems are in contact, the entropy transfer from warmer system is equal to the
entropy transfer to the colder system since the boundary has no thickness and
occupies no volume.
Note that work is entropy‐free, and no entropy is transferred with work.

Mass Flow: mass contains entropy as well as energy, both entropy and energy contents of
a system are proportional to the mass. When a mass in the amount of m enters or leaves a
system, entropy in the amount of ms (s is the specific entropy) accompanies it.

Entropy Balance for a Closed System


A closed system includes no mass flow across its boundaries, and the entropy change is
simply the difference between the initial and final entropies of the system.
The entropy change of a closed system is due to the entropy transfer accompanying heat
transfer and the entropy generation within the system boundaries:
Entropy change of the system = Entropy transfer with heat + Entropy generation
𝑄𝑘
𝑆2 − 𝑆1 = ∑ + 𝑆𝑔𝑒𝑛
𝑇𝑘
Therefore, for an adiabatic closed system, we have:
∆𝑆𝑎𝑑𝑖𝑎𝑏𝑎𝑡𝑖𝑐 = 𝑆𝑔𝑒𝑛
For an internally reversible adiabatic process ΔS = 0, because Sgen= 0.
The total entropy generated during a process can be determined by applying the entropy
balance to an extended system that includes both the system and its immediate
surroundings where external irreversibility might be occurring.

Sample Problem: Entropy balance for a closed system


Saturated liquid water at 100 C is contained in a piston‐cylinder assembly. The
water undergoes a process to the corresponding saturated vapor state, during which the
piston moves freely in the cylinder. There is no heat transfer with the surroundings. If the
change of state is brought about by the action of a paddle wheel, determine the network per
unit mass, in kJ/kg, and the amount of entropy produced per unit mass, in kJ/kg.K.

Assumptions:
1‐ The water in the piston‐cylinder assembly is a closed system.
2‐ There is no heat transfer with the surroundings.
3‐ The system is at an equilibrium state initially and finally. ΔPE = ΔKE = 0.

Solution
The network can be calculated by using the law:
ΔU + ΔKE + ΔPE = Q – W
That is simplifies to: ΔU = ‐ W
On a unit mass basis, the energy balance becomes:
W / m = ‐ (ug – uf)
W / m = ‐ 2087.6 kJ/kg
The negative sign indicates that the work input by the stirring is greater than the work done
by the water as it expands.
Using an entropy balance, the amount of entropy produced can be found. Since there is no
heat transfer,
2
𝛿𝑄
∆𝑆 = ∫ + 𝑆𝑔𝑒𝑛 = 𝑆𝑔𝑒𝑛
1 𝑇

On a unit mass basis, this becomes:


Sgen / m = sg ‐ sf
Sgen / m = 6.048 kJ / kg.K
Entropy Balance for a Control Volume
In addition to methods discussed for closed system, the entropy can be exchanged
through mass flows across the boundaries of the control volume.

The entropy balance in the rate form for a control volume becomes:
𝑑𝑆𝐶𝑉 𝑄𝑘°
= ∑ + ∑ 𝑚𝑖° 𝑠𝑖 − ∑ 𝑚𝑒° 𝑠𝑒 + 𝑆𝑔𝑒𝑛,𝐶𝑉
°
𝑑𝑡 𝑇𝑘
For a steady‐state steady‐flow process, it simplifies to:
𝑄𝑘°
°
𝑆𝑔𝑒𝑛,𝐶𝑉 = ∑ 𝑚𝑒° 𝑠𝑒 − ∑ 𝑚𝑖° 𝑠𝑖 − ∑
𝑇𝑘
Sample Problem: Entropy balance for a CV
Steam enters a turbine with a pressure of 3 MPa, a temperature of 400 °C, and a velocity
of 160 m/s. Saturated vapor at 100 °C exits with a velocity of 100 m/s. At steady‐state, the
turbine develops work equal to 540 kJ/kg. Heat transfer between the turbine and
its surroundings occur at an average outer surface temperature of 350 K. Determine the
rate at which entropy is produced within the turbine per kg of steam flowing, in
kJ/kg.K. Neglect the change in potential energy between inlet and exit.

Assumptions:
1‐ Steady state operation in CV. ΔPE = 0.
2‐ Turbine outer surface is at a specified average temperature.

Solution
From the mass balance, we know that m° = m°1 = m°2
Since the process is steady‐state, one can write:
𝑄𝑘° °
0= ∑ + 𝑚° (𝑠𝑖 − 𝑠𝑒 ) + 𝑆𝑔𝑒𝑛,𝐶𝑉
𝑇𝑘
The heat transfer occurs at Tb = 350 K, the first term of the right hand side of the entropy
balance reduces to Q°/ Tb
°
𝑆𝑔𝑒𝑛,𝐶𝑉 𝑄𝑘°
= − ° + (𝑠𝑖 − 𝑠𝑒 )
𝑚° 𝑚 𝑇𝑘
We need to calculate the rate of heat transfer. The first law (energy balance) can be used
to find the heat transfer rate. Combining the mass balance and the first law, one finds:
° °
𝑄𝐶𝑉 𝑊𝐶𝑉 𝑉22 − 𝑉12
= + (ℎ2 − ℎ1 ) + ( )
𝑚° 𝑚° 2
From the table, h1 = 3230.9 kJ/kg, and h2 = 2676.1 kJ/kg. After substitution, and
converting the units, one finds:
°
𝑄𝐶𝑉
= −22.6 𝑘𝐽/𝑘𝑔
𝑚°
From the table, s2 = 7.3549 kJ/kg.K and s1 = 6.9212 kJ/kg.K. Inserting values into
the expression for entropy production:
°
𝑆𝑔𝑒𝑛,𝐶𝑉 𝑄𝑘°
= − + (𝑠𝑖 − 𝑠𝑒 ) = 0.4983 𝑘𝐽/𝑘𝑔 𝐾
𝑚° 𝑚° 𝑇𝑘

Entropy
Entropy can be viewed as a measure of molecular disorder, or molecular randomness. As
a system becomes more disordered, the positions of the molecules become less
predictable and the entropy increases.

Figure 1: Entropy of a substance (level of disorder) increases when it melts from


solid phase to liquid phase. Ssolid < Sliquid < Sgas

Work is an organized form of energy, free of disorder or randomness, thus free of


entropy. Therefore, there is no entropy associated with energy transfer as work.
The quantity of energy is always preserved during an actual process, based on the first law,
but the quality is bound to decrease (the second law).
Processes can occur only in the direction of increased overall entropy or molecular
disorder. Thus, the entire universe is getting more and more chaotic every day.
At absolute zero (0 K), molecules become completely motionless, this represents a state of
ultimate molecular order (and minimum energy). Therefore, the entropy of a pure
crystalline substance at zero temperature is zero. That is because; there is no uncertainty
about the state of the molecules at that instant. This statement is the third law of
thermodynamics.
Since there is a reference for entropy (absolute zero), entropy is an absolute property. The
entropy measured with respect to absolute zero is called absolute entropy.
The two diagrams used most extensively in the second‐law analysis are the T‐s and h‐s
diagrams. For an internally reversible process, one can write:
𝛿𝑄𝑖𝑛𝑡,𝑟𝑒𝑣 = 𝑇𝑑𝑆 (𝑘𝐽)
Figure 2: On a T‐s diagram, the area under an internally reversible process presents
the heat transfer for the process.
For an internally reversible isothermal process, we have:
𝑄𝑖𝑛𝑡,𝑟𝑒𝑣 = 𝑇𝑜 𝑑𝑠
In a T‐s diagram, an isentropic process is represented by a vertical line. An
isentropic process is a process in which entropy remains constant. As a result an isentropic
process involves no heat transfer. Therefore:
Isentropic process (s2 = s1) = Reversible + Adiabatic

Entropy Change of Ideal Gas


The entropy change of an ideal gas can be obtained, by substituting du = cvdT and P = RT/v
into Gibbs equation.
𝑑𝑇 𝑑𝑣
𝑑𝑆 = 𝑐𝑣 +𝑅
𝑇 𝑣
2
𝑑𝑇 𝑣2
𝑠2 − 𝑠1 = ∫ 𝐶𝑣 (𝑇) + 𝑅 ln
1 𝑇 𝑣1

Assuming averaged values for specific heats, one obtains:


𝑇2 𝑣2 𝑘𝐽
𝑠2 − 𝑠1 = 𝐶𝑣,𝑎𝑣𝑒 ln + 𝑅 ln
𝑇1 𝑣1 𝑘𝑔 . 𝐾
𝑇2 𝑃2 𝑘𝐽
𝑠2 − 𝑠1 = 𝐶𝑝,𝑎𝑣𝑒 ln − 𝑅 ln
𝑇1 𝑃1 𝑘𝑔 . 𝐾

For isentropic processes of ideal gases, the following relationships can be found by setting
ds = 0,
𝑇2 𝑅 𝑣2
ln = − ln
𝑇1 𝐶𝑣 𝑣1
𝑇2 𝑣2 𝑅 𝑇2 𝑣1 𝑘−1
ln = ln( )𝐶𝑣 𝑜𝑟 ( ) = ( ) 𝑖𝑠𝑒𝑛𝑡𝑟𝑜𝑝𝑖𝑐 𝑝𝑟𝑜𝑐𝑒𝑠𝑠
𝑇1 𝑣1 𝑇1 𝑣2

Since R = cp – cv, k = cp / cv, and thus R / cv = k – 1.


In a similar manner, one finds:
𝑇2 𝑣1 𝑘−1
( )=( ) 𝑖𝑠𝑒𝑛𝑡𝑟𝑜𝑝𝑖𝑐 𝑝𝑟𝑜𝑐𝑒𝑠𝑠
𝑇1 𝑣2
These equations can be expressed in the following compact forms:
Tvk ‐1 = constant
TP(1 – k) / k = constant
Pvk = constant
The specific ratio k, varies with temperature, and in isentropic relations above an average
k value should be used.

Sample Problem: Isentropic process of ideal gas


A rigid, well‐insulated tank is filled initially with 5 kg of air at pressure 500 kPa
and a temperature 500 K. A leak develops, and air slowly escapes until the pressure of the
air remaining in the tank is 100 kPa. Using the ideal gas model, determine the
amount of mass remaining in the tank and its temperature.

Assumptions:
1‐ As shown in the figure, the closed system is the mass initially in the tank that remains
in the tank.
2‐ There is no significant heat transfer between the system and its surroundings.
3‐ Irreversibilities within the tank can be ignored as the air slowly escapes.

Solution
Using the ideal gas equation of state, the mass initially in the tank that remains in the tank
at the end of process is:
𝑃2 𝑉
𝑚2 = 𝑅𝑇 𝑃2 𝑇1
2
} 𝑚2 = ( ) ( ) 𝑚1
𝑃1 𝑉 𝑃1 𝑇2
𝑚1 = 𝑅𝑇
1
Since the volume of the tank V remains constant during the process. We need to find the
final temperature T2. For the closed system under consideration (m1), there are no
irreversibilities, and no heat transfer. Accordingly, it is an isentropic process, and thus the
isentropic relationships can be used:

With a constant k = 1.4 for air, after substituting values, one finds:
T2 = 315.55 K
Finally, inserting values into the expression for system mass
m2 = (100/500) (500/315.55) (5 kg) = 1.58 kg

REFERENCES

Athena.ecs.csus.edu. (2016). Notes on the Second Law. [online] Available at:


http://athena.ecs.csus.edu/~reardonf/ThermoNotesI/secndlaw.htm
Bahrami, M. (2015). Entropy. [online] Available at:
https://www.sfu.ca/~mbahrami/ENSC%20388/Notes/Entropy.pdf [Accessed 23
May 2018].
Borgnakke., Moran, M., Shapiro, H., Boettner, D. and Bailey, M. (2010). Fundamentals
of Engineering Thermodynamics. 6th ed. John Wiley & Sons Canada, Limited.
Nptel.ac.in. (2015). Objectives_template. [online] Available at:
http://nptel.ac.in/courses/112104113/lecture6/6_5.htm
Physicscatalyst.com. (2010). Thermodynamics Solved examples. [online] Available at:
https://physicscatalyst.com/heat/thermodynamics_solexam.php
Mpoweruk.com. (2014). Heat Engines. [online] Available at:
https://www.mpoweruk.com/heat_engines.htm
Physics LibreTexts. (2011). 15.5: Applications of Thermodynamics: Heat Pumps and
Refrigerators. [online] Available at:
https://phys.libretexts.org/?title=TextBooks_%26_TextMaps/College_Physics_Te
xtMaps/Map:_College_Physics_(OpenStax)/15:_Thermodynamics/15.5:_Applicat
ions_of_Thermodynamics:_Heat_Pumps_and_Refrigerators
Sanfoundry. (2018). Entropy Principles & Applications - Thermodynamics Questions and
Answers - Sanfoundry. [online] Available at: https://www.sanfoundry.com/
thermodynamics-questions-answers-entropy-principle-applications-1/ [Accessed
23 May 2018].
THERMODYNAMICS II
Lecture Manual

________________________________________________

PART III

Thermodynamic Properties of Fluid


Property Relations for Homogeneous Phases
Residual Properties
Residual Properties by Equation of State
Two-Phase System
Thermodynamic Diagrams and Tables
Generalized Property Correlations for Gases
THERMODYNAMIC PROPERTIES OF FLUIDS
Initial purpose in this chapter is to develop from the first and second laws the
fundamental property relations which underlie the mathematical structure of
thermodynamics. From these, we derive equations which allow calculation of enthalpy
and entropy values from PVT and heat-capacity data. We then discuss the diagrams
and tables by which property values are presented for convenient use. Finally, we
develop generalized correlations which provide estimates of property values in the
absence of complete experimental information.

Property Relations for Homogeneous Phases

The first law for a closed system of n moles is:

For the special case of a reversible process,

Equations from previous chapters are here written:

Together these three equations give:

where U, S, and V are molar values of the internal energy, entropy, and volume.
This equation, combining the first and second laws, is derived for the special case of a
reversible process.

Immediate application of these equations is to one mole (or to a unit mass) of a


homogeneous fluid of constant composition. For this case, they simplify to:

These fundamental property relations are general equations for a homogeneous


fluid of constant composition.

Maxwell's equations
Enthalpy and Entropy as Functions of T and P:

Consider first the temperature derivatives. The equation below defines the heat
capacity at constant pressure:

Combination of this equation gives:

The pressure derivative of the entropy results to the equation:

The corresponding derivative for the enthalpy is found to be:

The equation then becomes:

The functional relations chosen here for H and S are:

whence
The partial derivatives in these two equations are given by:

These are general equations relating the properties of homogeneous fluids of


constant composition to temperature and pressure.

Internal Energy as a Function of P

The pressure dependence of the internal energy is obtained by differentiation of


the equation, U=H-PV:

The Ideal-Gas State:


The coefficients of dT and dP are evaluated from heat-capacity and PVT data.
The ideal-gas state provides an example of PVT behavior:

The equation then reduces to:

where superscript "ig" denotes an ideal-gas value. These are merely restatements of
equations for ideal gases.
Alternative Forms for Liquids
Alternative form Equations result when ( V/ T)p is replaced by βV:

Also, if, in addition ( V/ P)T is replaced by –kV

These equations, incorporating β and k, although general, are usually


applied only to liquids. However, for liquids not near the critical point, the volume
itself is small, as are β and K. Thus at most conditions pressure has little effect on
the properties of liquids.
When (V/ T)p is replaced in favor of the volume expansivity, they become:

Since β and k are weak functions of pressure for liquids, they are usually
assumed constant at appropriate average values for integration.

The Gibbs Energy as a Generating Function


The fundamental property relations for homogeneous fluids of constant
composition given show that each of the thermodynamic properties U, H, A, and G
is functionally related to a special pair of variables. In particular,
dG= VdP-SdT

The Gibbs energy when given as a function of T and P therefore serves as a generating
function for the other thermodynamic properties, and implicitly represents complete
property information.

Residual Properties

Residual Gibb`s Energy:

Residual Volume

Fundamental property relation for residual properties


Residual Entropy

General expressions for Hig and Sig

IV. TWO-PHASE SYSTEM S


The curves shown in Fig. 1 represents phase
boundaries for a pure substance. A phase transition
at constant temperature and pressure occurs
whenever one of these curves is crossed, and as a
result the molar or specific values of the extensive
thermodynamic properties change abruptly. Thus,
the molar or specific volume of a saturated liquid
is very different from the molar or specific volume
of saturated vapor at the same T and P. This is true
as well for internal energy, enthalpy, and entropy. Figure 1. PT diagram for a pure substance
The exception is the molar specific Gibbs energy, which for a pure species does not change
during a phase transition such as melting, vaporization, or sublimation.
For two phases α and β of a pure species coexisting at equilibrium,
Gα = Gβ (1)
α β
Where G and G – molar Gibbs energies of the individual phases
For two phases coexist in equilibrium when the temperature is changed,
dGα = dGβ (2)
α α β β
V dP – S dT = V dP – S dT
sat sat
(3)
β α αβ
dP sat
S –S ∆S (4)
= β α= αβ
dT V –V ∆V
The entropy change ∆S and the volume change ∆Vαβ occur when a unit amount
αβ

of pure chemical species is transferred from phase α to phase β at the equilibrium pressure
and temperature.
At equilibrium pressure and temperature upon integration of equation,
dH = TdS – VdP (5)
αβ αβ
leads to latent heat of phase transition: ∆H = T∆S (6)
αβ αβ
Also equivalent to: ∆S = ∆H /T (7)
αβ
Then, dP sat
∆H
=
dT T∆Vαβ (8)
which is the Clapeyron equation,
For the particularly important case of phase transition from liquid l and v,
dPsat ∆Hlv
=
dT T∆Vlv (9)

Example 1: For vaporization at low pressures, one may introduce reasonable approximations
by assuming that the vapor phase is an ideal gas and that the molar volume of the liquid is
negligible compared with the molar volume of the vapor. How do these assumptions alter the
Clapeyron equation?
Solution: ∆Vlv = Vv = RT/Psat

dPsat ∆Hlv
=
dT T∆Vlv

dPsat ∆Hlv
=
dT RT2/Psat

dPsat/Psat ∆Hlv
=
dT/T2 R

∆Hlv = -R d ln Psat
d(1/T) (10)

Clausius-Clapeyron Equation
 Relates the latent heat of vaporization directly to the vapor-pressure curve.
 ∆Hlv is almost constant, virtually independent of T.
 Not valid, ∆Hlv decreases monotonically with increasing temperature from the triple
point to the critical point, where it becomes zero.
 The equation is applicable only at low pressures.

Temperature Dependence of the Vapor Pressure of Liquids


The Clapeyron equation is an exact thermodynamic relation, providing a vital
connection between the properties of different phases. A plot of ln Psat vs. 1/T generally yields
a line that is nearly straight:
ln Psat = A – B/T (11)
where A and B are constants for a given species.
Another equation which is very useful is the Antoine’s equation.
B
Antoine’s equation: ln Psat = A − T+C (12)
A principal advantage of this equation is that values off the constants A, B, and C
are readily available for a large number of species.
For accurate representation of vapor-pressure data over a wide temperature range requires an
equation of greater complexity which is Wagner’s equation:
Aτ + Bτ1.5 + Cτ3 + Dτ6
ln Psat = (13)
1–τ
τ = 1 – Tr
Other equation include:
Riedel’s Equation:
∆Hlv 1.092 (ln Pc – 1.013)
R Tn = 0.930 – Trn (14)
Where: Tn = normal boiling point
∆Hlv = latent heat of vaporization
Pc = critical pressure
Trn = residual temperature at Tn
Two Phase Liquid/Vapor Systems
When a system consists a saturated-liquid and saturated vapor phases coexisting in
equilibrium, the total value of any extensive property of the two-phase system is the sum of
the total properties of the phases. Written for this relation:
M = (1 – xv) Ml + xv Mv= Ml + xv ∆Mlv (15)
l v
where x and x , fractions of the total systems that are liquid and vapor called quality
M represents thermodynamic property (V, U, H, S, etc.)

V. THERMODYNAMIC DIAGRAMS AND TABLES


A thermodynamic diagram is a graph showing for a particular substance a set of
properties. The most common diagrams are: TS, PH (usually ln P vs. H) and HS (called a
Mollier diagram). Thermodynamic diagrams usually show a net of five different lines:
 Isobars - lines of constant pressure
 Isotherms – lines of constant temperature
 Dry adiabats – lines of constant potential temperature representing the temperature of
a rising parcel of dry air
 Saturated adibats or pseudoadiabats – lines representing the temperature of a rising
parcel saturated with water vapour
 Mixing ratio – lines representing the dew point of a rising parcel

Figure 1. PH diagram Figure 2. TS diagram

Figure 3. Mollier diagram


In many instances thermodynamic properties are tabulated. This has the advantage
that data can be represented more precisely than in diagrams, but the need for interpolation
is introduced. The steam tables are the most thorough compilation of properties for any
single material. However, tables are available for a number of other substances.

Example 2: Superheated steam originally at P1 and T1 expands through a nozzle to an


exhaust pressure P2. Assuming the process is reversible and adiabatic, determine the
downstream state of the steam and ∆H for the following conditions:
a. P1 = 1,000 kPa, t1 = 2500C, and P2 = 200kPa
b. P1 = 150 psia, t1 = 5000F, and P2 = 50 psia
Solution:
From the steam table,
At P1 =1000 kPa, T1 = 250°C, H1 = 2942.9 kJ/kg, S1 = 6.9252 kJ/kg.K
At P2 = 200 kPa, S2 = S1 = 6.9252 kJ/kg
Sl2 = 1.530 kJ/kg.K
Sv2 = 7.1268 kJ/kg.K

S2 = (1 – xv2) Sl2 + xv2 Sv2


6.9252 = 1.530 (1 – xv2) + 7.1268 xv2
xv2 = 0.9640

H2 = (1 – xv2) Hl2 + xv2 Hv2


H2 = (1 – 0.9640)(504.7) + (0.9640)(2706.3)
H2 = 2627.0 kJ/kg

∆H = H2 - H1
∆H = 2627.0 – 2942.9
∆H = -315.9 kJ/kg

VI. GENERALIZED PROPERTY CORRELATIONS FOR GASES


Compressibility factor, usually defined as Z = pV/RT, is unity for an ideal gas. In
most engineering work, the compressibility factor is used as a correction factor to ideal
behavior. Thus, Vreal = Z · Videal is used to calculate the actual volume, Vreal, as the product
of the compressibility factor and the ideal gas volume, all at the same pressure and
temperature. Z is most commonly found from a generalized compressibility factor chart as
a function of the reduced pressure, and the reduced temperature, where Pr and Tr are the
reduced variables and the subscript 'c' refers to the critical point.

Reduced Parameters
𝑇
𝑇𝑟 =
𝑇𝑐
𝑃
𝑃𝑟 =
𝑃𝑐
𝑉
𝑉𝑟 =
𝑉𝑐
Pitzer Correlations

𝑍 = 𝑍 ° + 𝜔𝑍1
Perry’s HB Eq. 2-63
The values for Z ⁰ and Z for gases are listed in the Perry’s HB Table 2-351 and 2-352,
1

respectively.

The Lee-Kesler correlation provides reliable results for gases which are nonpolar or only
slightly polar. When applied to highly polar gases, large errors can be expected.

Of the two kinds of data needed for evaluation of thermodynamic properties, heat
capacities and PVT data, the latter are most frequently missing. Fortunately, the
generalized methods developed for the compressibility factor are also applicable to residual
properties.
𝑃 = 𝑃𝑐 𝑃𝑟
𝑑𝑃 = 𝑃𝑐 𝑑𝑃𝑟
𝑇 = 𝑇𝑐 𝑇𝑟
𝑑𝑇 = 𝑃𝑐 𝑑𝑇𝑟
The resulting equations are
𝑃𝑟
𝐻𝑅 2
𝜕𝑍 𝑑𝑃
= −𝑇𝑟 ∫ ( )
𝑅𝑇𝑐 0 𝜕𝑇𝑟 𝑃𝑟 𝑃𝑟
𝑃𝑟 𝑃𝑟
𝑆𝑅 𝜕𝑍 𝑑𝑃 𝑑𝑃
= −𝑇𝑟 ∫ ( ) − ∫ (𝑍 − 1)
𝑅 0 𝜕𝑇𝑟 𝑃𝑟 𝑃𝑟 0 𝑃𝑟

Calculated values of the quantities as determined by Lee and Kesler are given as functions
of Tr. and Pr.
𝐻 𝑅 (𝐻 𝑅 )° (𝐻 𝑅 )1
= +𝜔
𝑅𝑇𝑐 𝑅𝑇𝑐 𝑅𝑇𝑐
𝑅 𝑅
(𝑆 ) ° (𝑆 𝑅 )1
𝑆
= +𝜔
𝑅 𝑅 𝑅

However, the generalized second-virial-coefficient correlation valid at low pressures forms


the basis for analytical correlations of the residual properties. The equation relating B to
the functions B0 and B1 is derived

Pitzer Correlations for Second Virial Coefficient


The generalized compressibility-factor correlation gave approximate
analysis expressions for limited range of pressures.
𝐵𝑃 𝑃𝑟
𝑍 = 1+ =1+Ḃ
𝑅𝑇 𝑇𝑟
𝐵𝑃𝑐
Ḃ=
𝑅𝑇𝑐
Second-virial-correlation yields the values for Ḃ:
Ḃ = 𝐵 ° + 𝜔𝐵1
Combining the two equations together,
𝐵𝑃𝑐
= 𝐵 ° + 𝜔𝐵1
𝑅𝑇𝑐
𝑃𝑟 𝑃𝑟
𝑍 = 1 + 𝐵 ° + 𝜔𝐵1
𝑇𝑟 𝑇𝑟
Perry’s HB Eq. 4-73

Since B, B⁰, and B1 are functions of T only,


𝑃𝑐 𝑑𝐵 𝑑𝐵 ° 𝑑𝐵1
= +𝜔
𝑅𝑇𝑐 𝑑𝑇𝑟 𝑑𝑇𝑟 𝑑𝑇𝑟

Calculated values of the enthalpies and entropies are given as functions of Tr.
𝐻𝑅 𝑃 𝑑𝐵
= (𝐵 − 𝑇𝑟 )
𝑅 𝑅 𝑑𝑇𝑟
𝑆𝑅 𝑃 𝑑𝐵
=
𝑅 𝑅𝑇𝑐 𝑑𝑇𝑟
This equations result to:
𝐻𝑅 𝑑𝐵 ° 𝑑𝐵1
= 𝑃𝑟 (𝐵 ° − 𝑇𝑟 + 𝜔 (𝐵1 − 𝑇𝑟 ))
𝑅𝑇𝑐 𝑑𝑇𝑟 𝑑𝑇𝑟
𝑆𝑅 𝑑𝐵 ° 𝑑𝐵1
= 𝑃𝑟 ( +𝜔 )
𝑅 𝑑𝑇𝑟 𝑑𝑇𝑟
The dependence of B⁰ and B1 on reduced temperature is given by
0.422
𝐵 ° = 0.083 − 1.6
𝑇𝑟
Perry’s HB Eq. 4-77
0.179
𝐵1 = 0.139 − 4.2
𝑇𝑟
Perry’s HB Eq. 4-78

Correlations for the Third Virial Coefficient


Accurate data for third virial coefficients are far less common than for second virial
coefficients.
𝑍 = 1 + 𝐵𝜌 + 𝐶𝜌2
Equation for third virial coefficient is a function of ρ, molar density, = 1/V.
𝑃𝑟 𝑃𝑟 2
𝑍 =1+Ḃ + Ċ( )
𝑇𝑟 𝑍 𝑇𝑟 𝑍
Perry’s HB Eq. 4-94
𝐶𝑃𝑐 2
Ċ= 2 2
𝑅 𝑇𝑐
Pitzer-type correlation for Ċ can be written as
Ċ = 𝐶 ° + 𝜔𝐶 1
An expression for C⁰ as a function of reduced temperature is given by Orbey and Vera
0.02432 0.00313
𝐶 ° = 0.01407 + −
𝑇𝑟 𝑇𝑟 10.5
Perry’s HB Eq. 4-98
0.05539 0.00242
𝐶 1 = −0.02676 + −
𝑇𝑟 2.7 𝑇𝑟 10.5
Perry’s HB Eq. 4-99
Calculation for path changes ΔH and ΔS
The generalized correlations for HR and SR , together with ideal-gas heat capacities, allow
calculation of enthalpy and entropy values of gases at any temperature and pressure.

Three Step Calculation Path


The actual path from state 1 to state 2
(dashed line) is replaced by a three-step
calculational path:

Step 1real to 1ideal: A hypothetical


process that transforms a real gas into
an ideal gas at T1 and P1. The enthalpy
and entropy changes for this process
are:
𝑖𝑔
𝐻1 − 𝐻1 = −𝐻1𝑅
𝑖𝑔
𝑆1 − 𝑆1 = −𝑆1𝑅
Step 1ideal to 2ideal: Changes in the ideal-
gas state from (T1, P1) to (T2, P2). For
this process,
𝑖𝑔 𝑖𝑔
∆𝐻 = 𝐻2 − 𝐻1
𝑇2
𝑖𝑔
∆𝐻 = ∫ 𝐶𝑝 𝑑𝑇
𝑇1
𝑖𝑔 𝑖𝑔
∆𝑆 = 𝑆2
− 𝑆1
𝑇2
𝑖𝑔 𝑑𝑇 𝑃2
∆𝑆 = ∫ 𝐶𝑝 − 𝑅 ln
𝑇1 𝑇 𝑃1
Step 2ideal to 2real: Another hypothetical process that transforms the ideal gas back into a
real gas at T2 and P2. Here,
𝑖𝑔
𝐻2 −𝐻1 = 𝐻2𝑅
𝑖𝑔
𝑆2 −𝑆1 = 𝑆2𝑅
For a change from state 1 to state 2,
𝑇1
𝑖𝑔 𝑖𝑔
𝐻1 = 𝐻0 + ∫ 𝐶𝑝 𝑑𝑇 + 𝐻1𝑅
𝑇0
𝑇2
𝑖𝑔 𝑖𝑔
𝐻2 = 𝐻0 + ∫ 𝐶𝑝 𝑑𝑇 + 𝐻2𝑅
𝑇0
The enthalpy change for the process, ΔH = H2 – H1, is the difference between these two
equations:
𝑇2
𝑖𝑔
∆𝐻 = ∫ 𝐶𝑝 𝑑𝑇 + 𝐻2𝑅 − 𝐻1𝑅
𝑇1
Similarly for entropy change,
𝑇2
𝑖𝑔 𝑑𝑇 𝑃2
∆𝑆 = ∫ 𝐶𝑝 − 𝑅 ln + 𝑆2𝑅 − 𝑆1𝑅
𝑇1 𝑇 𝑃1
Written alternative form these equations become:
𝑖𝑔
∆𝐻 = (𝐶𝑝 ) (𝑇2 − 𝑇1 ) + 𝐻2𝑅 − 𝐻1𝑅
𝐻
𝑖𝑔 𝑇2 𝑃2
∆𝑆 = (𝐶𝑝 ) ln − 𝑅 ln + 𝑆2𝑅 − 𝑆1𝑅
𝑆 𝑇1 𝑃1
Gas Mixtures
Although no theoretical basis exists for extension of generalized correlations to mixtures,
approximate results for mixtures can often be obtained with pseudocritical parameters
resulting from simple linear mixing rules according to the definitions:
𝜔 = ∑ 𝑦𝑖 𝜔𝑖
𝑖

𝑇𝑝𝑐 = ∑ 𝑦𝑖 𝑇𝑐𝑖
𝑖

𝑃𝑝𝑐 = ∑ 𝑦𝑖 𝑃𝑐𝑖
𝑖
The values so obtained are the mixture and pseudocritical temperature and pressure, Tpc
and Ppc which replace Tc, and Pc, to define pseudoreduced parameters:
𝑇
𝑇𝑝𝑟 =
𝑇𝑝𝑐
𝑃
𝑃𝑝𝑟 =
𝑃𝑝𝑐

Reference:
Smith, J.M. et al., Introduction to Chemical Engineering Thermodynamics 5th Edition
McGraw-Hill Companies, Inc, Singapore, 2001.
THERMODYNAMICS II
Lecture Manual

________________________________________________

PART IV

Application of Thermodynamics to Flow Processes


Duct Flow of Compressible Fluids
Turbines (Expanders)
Compression Processes
Production of Power from Heat
Steam Power Plant
Internal Combustion Engines
Refrigeration
Vapour Compression Cycle
Choice of Refrigerant
Heat Pump
Intended Learning Outcomes
1. Recognize the application of first law of thermodynamics in solving
thermodynamic processes.
2. Present the principles and applications of Thermodynamics in terms of pipe flow,
turbines, compression processes, gas power cycles, refrigeration and liquefaction.
3. Exercise analysis and answering problems pertaining to the Application of
Thermodynamics to Flow Processes and Steam Power Plant.

APPLICATION OF THERMODYNAMICS TO FLOW PROCESSES

Introduction
Flow processes inevitably result from pressure gradients within the fluid.
Moreover, temperature, velocity and even concentration gradients may exist within the
flowing fluid. This contrasts with the uniform conditions that prevail at equilibrium in
closed systems. The distribution of conditions in flow systems requires that properties be
attributed to point masses of fluid. Thus we assume that intensive properties, such as
density, specific enthalpy, specific entropy, etc., at a point are determined solely by the
temperature, pressure, and composition at the point, uninfluenced by gradients that may
exist at the point. Moreover, we assume that the fluid exhibits the same set of intensive
properties at the point as though it existed at equilibrium at the same temperature, pressure
and composition.

EQUATION OF BALANCES

 Net  rate  of   Net  rate  of  Time  rate  of 


     
transport  of  X    generation  of  X   change  of  X 
int o  control  volume within  control  volume within  control  volume
     

dX CV
X T  X G 
dt
Differential Equation of Balance
dX T  dX G  dX CV
dX CV
Where: dX T  X T dt dX G  X G dt dX CV  dt
dt

Integral Equation of Balance

X T  X G  X CV

t2 t2
Where: X T   X T dt X G   X G dt
t1 t1

t2 dX CV
X CV   dt  X CV (t 2 )  X CV (t1 )
t1 dt

For flow processes with no chemical reaction happening between the surrounding and the
system
dX total
X G  0
dt

Idealization for practical application of equation of balance:


1. Flow is presumed unidirectional at any cross section of a conduit where
thermodynamic, kinetic, and dynamic properties are assigned or evaluated, namely,
at entrances to and exits from the equipment under consideration.
2. At such a cross section these same properties are presumed not to vary in a direction
perpendicular to the direction of flow. Thus properties such as velocity,
temperature, and density, assigned or evaluated for the cross section, have values
which are appropriate averages over the cross section.

Mass Balance
dmCV
Mass is conserved, X G  0 m T 
dt
 T , net transport of mass into the control volume
Where: m
dmCV
 (m ) fs  0
dt
m  mass flow rate  uA

Where: ρ, average fluid density


u, average velocity
A, cross sectional area of the entrance or exit duct

dmCV
Continuity equation:  uA fs  0
dt

Steady state: ( uA) fs  0

 2u2 A2  1u1 A1  0
m  const   2u2 A2  1u1 A1

u1 A1 u 2 A2 uA
m   
V1 V2 V

Energy Balance
Considering transport energy balance:

X T  Q  W  [( PV )m ] fs  [(U  12 u 2  zg )m ] fs

Since U + PV = H

X T  Q  W  [( H  12 u 2  zg )m ] fs
Energy Balance Equation with accumulation:

d (mU ) CV
 ] fs  Q  W
 [( H  12 u 2  zg )m
dt

For most applications


d (mU ) CV
 ( Hm ) fs  Q  W
dt

Energy Balances for Steady State Flow Processes


General energy balance
[( H  12 u 2  zg )m ] fs  Q  W S
One entrance and one exit
( H  12 u 2  zg )m  Q  W S

Q W S
( H  12 u 2  zg )    Q  WS
m m

u 2
H   gz  Q  WS
2
Entropy Balance
dS
ST  SG  CV
dt
Entropy is transported across the control surface in two ways:
1. Transport by means of heat transfer.Net rate of entropy transport into the control
volume:
Q
T
j

j cs , j

2. Transport by flowing streams.


  ( Sm ) fs

Q
Entropy transport: ST    ( Sm ) fs
j Tcs , j

Substituting equation above

Q j d (mS ) CV
Tj
 ( Sm ) fs  SG 
dt
cs , j

Q j Q j  T , j  TCS , j 
T 
T , j
  Q j 
 T T


j cs , j j j   , j CS , j 

Q j d (mS ) CV
T  ( Sm ) fs  SG  S 'G 
dt
j ,j
 T , j  TCS , j 
Where: S 'G   Q j   external thermal irreversibilities
 T T 
j   , j CS , j 

SG  S 'G  SG,total


Q j d (mS ) CV
T  Sm  fs  SG ,total 
j ,j dt
Q j d (mS ) CV
T  ( Sm ) fs 
dt
j ,j

Limiting case of equality applies to a process that is completely reversible:


1. The process is internally reversible within the control volume.
2. Heat transfer external to the control volume is reversible.

For steady flow process


Q j
( Sm ) fs    SG ,total
j T , j

Qj
S    S G ,total
j T , j

DUCT FLOW OF COMPRESSIBLE FLUIDS


Thermodynamics does provide equations that interrelate the changes occurring in
pressure, velocity, cross-sectional area, enthalpy, entropy, and specific volume of a flowing
stream. We consider here the adiabatic, steady-state, one-dimensional flow of a
compressible fluid in the absence of shaft work and of changes in potential energy. The
pertinent thermodynamic equations are first derived; they are then applied to flow in pipes
and nozzles.

The appropriate energy balance is shown below. With Q, W, and Δz all set equal to zero,
u 2
H  0
2

Differential form dH  udu

uA
Continuity Equation )0 d(
V
dV du dA
  0
V u A

Equations derived
dV T V
 dS  2 dP
V Cp c
u 2 u2
(1  M 2 )VdP  (1  )TdS  dA  0
Cp A
Where: M is the Mach number, ratio of the speed of the fluid in the duct to the speed of
sound in the fluid.
 u 2 
 M2 
udu   TdS   1 u
2
Cp
   dA  0
1 M 2 1 M
2
 A
 
 
dP  u 2  dS u 2 dA
V (1  M )
2
 T 1    0
dx  Cp  dx A dx

 u 2 
 M2 
 T   dS   1  u dA  0
2
du Cp
u  
dx 1  M 2  dx  1  M 2  A dx
 
 
dS
0
dx

PIPE FLOW
Steady state adiabatic flow in a horizontal pipe of constant cross-sectional area, dA/dx=0.

 u 2 
1 
    dS
dP T Cp
dx V 1 M 2  dx
 
 
 u 2 
 M2 
 T   dS
du Cp
u  dx
dx 1 M 2
 
 
dP du
Subsonic flow, M2 < 1 0 and 0
dx dx

For subsonic flow, the maximum fluid velocity obtainable in a pipe of constant cross
section is the speed of sound, and this value is reached at the exit of the pipe. The equations
for pipe flow indicate that when supersonic stream is enters a pipe of constant cross section,
a compression shock occurs, the result of which is an abrupt and finite increase in pressure
and decrease in velocity to a subsonic value.

Sample Problem 1
Consider the steady-state, adiabatic, irreversible flow of an incompressible liquid in a
horizontal pipe of constant cross-sectional area. Show that:
a. The velocity is constant.
b. The temperature increases in the direction of flow.
c. The pressure decreases in the direction of flow.
Solution:

a.
u2A2 u1A1
=
𝑉2 𝑉1

A2 = A1 *Constant cross-sectional area


V2 = V1 *Incompressible fluid

Hence, u2 = u1

b.

𝑇2 𝑑𝑇
Sg = S2 = S1 = ∫𝑇1 𝐶 𝑇
But Sg is positive (flow is irreversible) and hence, by the last equation, T2 > T1, the
temperature increases in the direction of flow.

c.
As shown in a, u2 = u1, and therefore the energy balance reduces for the stated conditions
to H2-H1 = 0. Combining this with the integrated form applied to an incompressible liquid
yields:

𝑇2
H2 – H1 = ∫𝑇1 𝐶𝑑𝑇 + 𝑉 ( 𝑃2 − 𝑃1) = 0
Whence,
𝑇2
V (P2 – P1) = - ∫𝑇1 𝐶𝑑𝑇
Thus the equation, P2 < P1 and pressure decreases in the direction of flow.

NOZZLES
The limitations observed for flow of compressible fluids in pipes do not extend
properly designed nozzles, which bring about the interchange of internal and kinetic energy
of a fluid as the result of a changing cross-sectional area available for flow. The relation
between nozzle length and cross-sectional area is not susceptible to thermodynamics
analysis, but is a problem in fluid mechanics. In a properly designed nozzle, area changes
with length in such a way as to make the flow nearly frictionless. In the limit of reversible
flow, the rate of entropy increase approaches zero, and dS/dX = 0.

dP u 2  1  dA
  
dx VA  1  M 2  dx

du u  1  dA
  
dx A  1  M 2  dx

The characteristics of flow depend on whether the flow is subsonic (M < 1) or supersonic
(M > 1).
Characteristics of Flow for a Nozzle
Subsonic: M < 1 Supersonic: M > 1
Converging Diverging Converging Diverging
dA/dx - + - +
dP/dx - + + -
du/dx + - - +

( 1) / 
P2 2P1V1   P2  
u  u  2 VdP 
2 2
1    
  1   P1 
2 1
P1
 

 /( 1)
P2  2 
Pressure ratio at the throat:  
P1    1 

Sample Problem 2
A high-velocity nozzle is designed to operate with steam at 700 kPa and 300⁰C. At
the nozzle inlet velocity is 30 m/s. Calculate values of the ratio A/A1 (where A1 is the
cross-sectional area of the nozzle inlet) for the sections where the pressure is 600,500, 400,
300 and 200 kPa. Assume that the nozzle operates isentropically.

Solution:

𝐴 𝑢1 𝑉
= And u2 = u12 – 2 (H-H1)
𝐴1 𝑉1 𝑢

Initial values of entropy, enthalpy, and specific volume from the steam tables:

S = 7.29997 kJ/ kg K H = 3,020.4 x 103 J/kg V = 418.25 cm3/g

A 30 418.25
= x = 0.120
A1 371.39 3282.3

Area ratios for other pressure are evaluated the same way.

THROTTLING PROCESS

A throttling process occurs when a fluid passes through a restriction, partially


closed valve, or porous plug, with negligible change in kinetic and potential energy. If the
process is adiabatic, 1`the process is isenthalpic, such that H=0. For an ideal gas, the
process is not only isenthalpic but also isothermal. But for real system, the increase or
 T 
decrease of temperature is determined via the Joule-Thomson coefficient,     . If
 P  H
 is positive, the temperature will decrease as a result of the throttling process, and increase
if the coefficient is negative.
RT 2  Z 
  
C P P  T  P
Joule/Thomson inversion curve, define at condition (Z / T ) P  0 for which  =0, obtain
for real gases, separates the region of positive  from that of negative  .

Sample Problem 3
Propane gas at 20 bar and 400 K is throttled in a steady-state flow process to 1 bar.
Estimate the final temperature of the propane and its entropy change. Properties of propane
can be found from suitable generalized correlations.

Solution:

ΔH = (Cpig)H ( T2 – T1) + H2R – H1R = 0


H R
T2 = (Cp1ig)H + T1
For propane, Tc= 369.8 K Pc = 42.48 bar ω = 0.152

H1R = (8.314)(369.8)(-0.452) = -1,390 J/mol


The only remaining quantity to be evaluated is (Cpig)H. Data for propane provide the heat
capacity equation:

(Cpig )
= 1.213 + 28.785 x 10 −3 𝑇 − 8.824 𝑥 10−6 𝑇 2
𝑅

(Cpig)H = 94.07 J/ mol K


T2 = (-1.390 / 94.07) + 400 = 385.2 K
Tam = (400+385.2)/2 = 392.6 K
(Cpig)H = 92.73 J/ mol K

𝑇2 𝑃2
ΔS = (Cpig)S ln 𝑇1 − 𝑅 ln 𝑃1 - 𝑆1 𝑅
(Cpig)S = (Cpig)H = 92.73 J/ mol K
J
S1 R = (8.314)(0.2934) = −2.439 mol K
ΔS = 92.73 ln (385/400) – 8.314 ln (1/20) +2.439 = 23.80 J/mol K

TURBINE
The expansion of a gas in a nozzle to produce a high-velocity stream is a process
that converts internal energy into kinetic energy, which in turn is converted into shaft work
when the stream impinges on blades attached to a rotating shaft. Thus a turbine (or
expander) consists of alternate sets of nozzles and rotating blades through which vapor or
gas flows in a steady-state expansion process. The overall result is the conversion of the
internal energy of a high-pressure stream into shaft work. When steam provides the motive
force as in a power plant, the device is called turbine; when it is a high-pressure gas, such
as ammonia or ethylene in a chemical or petrochemical plant, the device is called an
expander.
Steady-state flow through a turbine or expander

 Adiabatic Turbine
 Negligible potential and kinetic energy changes

Actual shaft work produced=  W s  H  H 1  H 2


maximum shaft work produced=  W sisentropic   H s  H 1  H 2 s
actual shaft work produced
  turbine efficiency 
max imum shaft work produced

W S H1  H 2
 

 W S isentropic  H1  H 2 S

Sample Problem 4
A steam turbine with rated capacity of 56,400 kW( 56,400 kJ/s) operates with steam
at inlet conditions of 8,600 kPa and 500⁰C, and discharges into a condenser at a pressure
of 10 kPa. Assuming a turbine efficiency of 0.75, determine the state of steam at discharge
and the mass rate of flow of the steam.

Solution:
At inlet conditions,
H1 = 3,391.6 kJ/kg S1= 6.6858 kJ/kg

If the expansion to 10 kPa is isentropic,then S2 = S1 = 6.6858

S2 = S2+ x2 (S2V – S2L)


6.6858 = 0.6493 +x2 ( 8.1511 – 0.6493)
X2 = 0.8047

H2 = H2 + x2 (H2V – H2L)
H2 = 191.8+ (0.8047)(2,584.8 – 191.8) = 2,1147.4 kJ/kg
ΔH = n(ΔH)s = 0.75(-1,274.2) = -955.6 kJ/kg
H2 = H1 + ΔH = 3,391.69 – 955.6 = 2,436.0 kJ/kg
Thus the steam in its actual final state is also wet, with its quality given by:
2,436.0 = 191.8 + x2 ( 2,584.8 – 191.8 )
X2 = 0.93378

COMPRESSOR

Steady-state Compression Process

 Adiabatic compressor
 Negligible potential and kinetic energy changes

actual shaft work consumed= W s  H  H 2  H 1

minimum shaft work consumed = W sisentropic   H s  H 2 S  H 1
min imum shaft work consumed
  compressor efficiency 
actual shaft work consumed

W S isentropic  H 2S  H1
 

WS H 2  H1

Sample Problem 5
Saturated-vapor steam at 100 kPa is compressed adiabatically to 300 kPa. If the
compressor efficiency is 0.75, what is the work required andwhat are the properties of the
discharge stream.

Solution:

For saturated steam at 100 kPa,


S1 = 7.3598 kJ/kg K H1 = 2.675.4 kJ/kg
*for isentropic compression to 300 kPa, S2 = S1

(ΔH)s = 2,888.8 – 2,675.4 = 213.4 kJ/kg


ΔH = (ΔH)s / n = 213.4 / 0.75 = 284.5 kJ/kg

H2 = H1 + ΔH = 2,675.4 + 284.5 = 2,959.9 kJ/kg


For saturated steam with this enthalpy, interpolation yields:

T2 =246.1 C S2 = 7.5019 kJ/ kg K

The work required is:

Ws = ΔH = 284.5 kJ/kg

PUMPS
Liquids are usually moved by pumps, generally rotating equipment. The same
equations apply to adiabatic pumps as to adiabatic compressors.

 Adiabatic pump
 Negligible potential and kinetic energy changes


actual shaft work consumed= W s  H  H 2  H 1

minimum shaft work consumed = W sisentropic   H s  H 2 S  H 1

min imum shaft work consumed


  pump efficiency 
actual shaft work consumed


W S isentropic   H 2S  H1

W H 2  H1
S

For incompressible liquids,



W sisentropic   H s  H 2 S  H 1  V1 P2  P1   V2 P2  P1 
Since it is not very easy to evaluate the thermodynamic properties of sub-cooled or
compressed liquids, the following approximations may be used

H  C p T  V 1  T P
T2
S  C p ln  VP
T1

Sample Problem 6
Water at 45 C and 10 kPa enters an adiabatic pump and is discharged at a pressure
of 8,600 kPa. Assume the pump efficiency to be 0.75. Calculate the work of the pump, the
temperature change of the water, and the entropy of the change of the water.

Solution:

The following are the properties of sat’d liquid at 45 C


V = 1.010 cm3/kg β = 425 x 10-6/K Cp = 4.178 kJ/kg K

Ws (isentropic) = (ΔH)s = (1,010)(8,600-10) = 8.676 x 106 kPa cm3/kg


Because 1 kJ = 106 kPa cm3
Ws (isentropic) = (ΔH)s = 8.676 kJ/kg

(ΔH)s 8.676
ΔH = = = 11.57 kJ/kg
𝑛 0.75
Ws = ΔH = 11.58 kJ/kg

The temperature change of the water during pumping

11.57 = 4.178 ΔT + 1.010 ( 1- (425 X 10-6)*318.15)(8.590/106)


ΔT = 0.97 K or 0.97 C

The entropy change of water is given by

ΔS = 4.178 ln ( 319.12/318/15) – (425 X 10-6)(1.010)(8.590/106) = 0.0090 kJ/kg K/

PRODUCTION OF POWER FROM HEAT


The steam power plant is a large-scale heat engine in which the working fluid
(H2O) is in steady state flow successively through a pump, a boiler, a turbine, and a
condenser in a cyclic process. The working fluid is separated from the heat source, and
heat is transferred across a physical boundary. In a fossil-fuel fired plant the combustion
gases are separated from the steam by boiler-tube walls.
The internal-combustion engine is another form of heat engine, wherein high
temperatures are attained by conversion of the chemical energy of a fuel directly into
internal energy within the work-producing device such as otto and diesel cycles and the
gas turbine.

Carnot Cycle
The Carnot cycle gives the most efficient heat engine that can operate between two fixed
temperatures TH and TL; it is independent of the type of working fluid and can be closed
or steady flow.

Area, which is enclosed by cyclic curve presents net work or heat transfer during the
cycle.
Process Description
1-2 Isothermal heat addition
2-3 Isentropic expansion
3-4 Isothermal heat rejection
4-1 Isentropic compression
𝑊𝑛𝑒𝑡
𝜂th = 𝑞𝑖𝑛

𝑞𝑖𝑛 − 𝑞𝑜𝑢𝑡
𝜂th = 𝑞𝑖𝑛

𝑇
𝜂th, Carnot = 1 - 𝑇 𝐿
𝐻

Ideal Rankine Cycle

 The ideal Rankine Cycle is internally reversible


 No friction effects
 Turbine and pump are reversible and adiabatic - Isentropic

The following are the processes of an ideal rankine cycle:


 1-2 Isentropic compression in pump
 2-3 Constant pressure heat addition in boiler
 3-4 Isentropic expansion in turbine
 4-1 Constant pressure heat rejection in condenser
Sample Problem 7
Compute the thermal efficiency of an ideal Rankine cycle for which steam leaves
the boiler as superheated vapor at 6 MPa, 350oC, and is condensed at 10 kPa.
Rankine Vapor Power Cycle
500

6000 kPa
400

3
300

T [C]
200

10 kPa
100 2
4
1
0
0 2 4 6 8 10 12

s [kJ/kg-K]

We use the power system and T-s diagram shown above.


P2 = P3 = 6 MPa = 6000 kPa
T3 = 350oC
P1 = P4 = 10 kPa

Using Steam Table


 kJ  kJ
 h1  h f  191.81 h3  3043.9
P1  10 kPa   kg 
P3  6000 kPa   kg
 
Sat. liquid   m3 T3  350 C  
o
kJ
v1  v f  0.00101 s  6.3357
 kg  3 kg  K

w pump  v1 ( P2  P1 )
m3 kJ
 0.00101 (6000  10) kPa 3
kg m kPa
kJ
 6.05
kg

Now, h2 is found from,


h2  wpump  h1
kJ kJ
 6.05  191.81
kg kg
kJ
 197.86
kg

The heat transfer per unit mass is,


Q
qin  in  h3  h2
m
kJ
 (3043.9  197.86)
kg
kJ
 2845.1
kg
We find the properties at state 4 from the steam tables by noting s4 = s3 = 6.3357 kJ/kg-K
and asking three questions.
kJ kJ
at P4  10kPa : s f  0.6492 ; sg  8.1488
kg  K kg  K
is s4  s f ?
is s f  s4  sg ?
is sg  s4 ?
s4  s f  x4 s fg
s4  s f 6.3357  0.6492
x4    0.758
s fg 7.4996

h4  h f  x4 h fg
kJ kJ
 191.81  0.758(2392.1)
kg kg
kJ
 2005.0
kg

The turbine work per unit mass is


wturb  h3  h4
kJ
 (3043.9  2005.0)
kg
kJ
 1038.9
kg

The net work done by the cycle is


wnet  wturb  wpump
kJ
 (1038.9  6.05)
kg
kJ
 1032.8
kg

The thermal efficiency is


kJ
1032.8
w kg
th  net 
qin 2845.1 kJ
kg
 0.363 or 36.3%

Gas Power Cycle


 Analysis of power cycles
 The working fluid remains a gas throughout the entire cycle.
 Neglect friction
 Neglect the required time for establishing the equilibrium state
 Air-Standard assumptions
 The working fluid is air, which continuously circulates in a closed loop and
always behaves as an ideal gas.
 All the processes that make up the cycle are internally reversible.
 The combustion process is replaced by a heat-addition process from an
external source.
 The exhaust process is replaced by a heat-rejection process that restores the
working fluid to its initial state.

Otto Cycle
The processes in Otto Cycle are as per following:
 1-2 Isentropic compression
 2-3 Constant volume heat addition
 3-4 Isentropic expansion
 4-1 Constant volume heat rejection

 Related formula based on thermodynamic properties


 Thermal efficiency of Otto Cycle is given as:
1
𝜂,Otto = 1 - 𝑟 𝑘−1

𝑉
Where r is the compression ratio r= 𝑉1
2
 Mean Effective Pressure

𝑊𝑛𝑒𝑡
MEP = 𝑉
𝑚𝑎𝑥 − 𝑉𝑚𝑖𝑛

Sample Problem 8
An Otto cycle having a compression ratio of 9:1 uses air as the working fluid. Initially P1
= 95 kPa, T1 = 17°C, and V1 = 3.8 liters. During the heat addition process, 7.5 kJ of heat
are added. Determine all T's, P's, 𝜂th, the back work ratio and the mean effective pressure.

Solution:
T1 = 290K
𝑉1
=9
𝑉2
Q23 = 7.5 kJ
P1 = 95 kPa
V1 = 3.8 Liters

Process 1-2 (isentropic compression)


𝑇2 𝑉 𝑘−1
= ( 𝑉1 )
𝑇1 2

T2 = 290(0.9)0.4 = 698.4K

𝑃2 𝑉 𝑘−1
= ( 𝑉1 )
𝑃1 2

P2 = 95(9)1.4 = 2059kPa

Process 2-3 (Constant volume heat addition)


Q23 = mCv(T3 – T2)

P1v1 = RT1

0.2871(290)
v1 = = 0.875 m3/kg
95

𝑄23 𝑣1
q23 = = Q23 = 1727 kJ/kg
𝑚 𝑉1

q23 = Cv (T3 – T2) = 0.718(T3 – 698.4)

T3 = 3103.7K

But V3 = V2

𝑃3 𝑃
= 𝑇2
𝑇3 2
P3 = 9.15 MPa

Process 3-4 (isentropic expansion)

𝑇4 𝑉 𝑘−1
= ( 𝑉3 )
𝑇3 4

T4 = T3(1/9)0.4 = 1288.8K

𝑃4 𝑉 𝑘
= ( 𝑉3 )
𝑃3 4

P4 = P3(1/9)1.4 = 422 kPa

Process 4-1(Constant Volume Heat Rejection)

Q41 = mCV(T4 – T1)


q41 = Cv(T4 – T1)
= 0.718(1288.8-290)
= 717.1 kJ/kg

Then,
Wnet = qin - qout = q23 – q41 = 1009.6kJ/kg
𝑊𝑛𝑒𝑡
𝜂,Otto = 𝑞𝑖𝑛
𝜼,Otto = 0.585 (58.5%)

𝑊𝑛𝑒𝑡 𝑊𝑛𝑒𝑡
MEP = 𝑉 = 1
𝑚𝑎𝑥 − 𝑉𝑚𝑖𝑛 𝑉1 − (1− )
𝑟
MEP = 1298kPa

Diesel Cycle
The processes in Diesel Cycle are as per following:
 1-2 Isentropic compression
 2-3 Constant pressure heat addition
 3-4 Isentropic expansion
 4-1 Constant volume heat rejection
 Related formula based on thermodynamic properties

 Thermal efficiency of Otto Cycle is given as:


1 𝑟 𝑘 −1
𝜂,Diesel = 1 - 𝑟 𝑘−1 (𝑘 (𝑟𝑐 −1))
𝑐

Sample Problem 9
An air-standard Diesel cycle has a compression ratio of 18 and a cut-off ratio of 2.5. The
state at the beginning of compression is fixed by P = 0.9 bar ant T = 300K. Calculate:
i. the thermal efficiency of the cycle,
ii. ii. the maximum pressure, Pmax, and
iii. iii. The mean effective pressure.

Solution:
Data given:
𝑉1
=18
𝑉2

𝑉3
=2.5
𝑉2

Process 1-2 (isentropic compression)


𝑇2 𝑉 𝑘−1
= ( 𝑉1 )
𝑇1 2
T2 = 300(18)0.4 = 953.3K

Process 2-3 (Constant Pressure heat addition)

P2= P3

𝑉2 𝑉
= 𝑇3
𝑇2 3

𝑉
T3 = T2 (𝑉3 ) = 2383.3K
2
Process 3-4 (isentropic expansion)

𝑉4 𝑉 𝑉
= 𝑉1 (𝑉2 ) = 18(1/2.5) = 7.2
𝑉3 2 3
𝑇4 𝑉3 𝑘−1
= (𝑉 )
𝑇3 4

T4 = 2383.3(1/7.2)0.4 = 1082K

Qin = Q23 = mCp(T3 – T2)


qin = Cp(T3 – T2) = 1437.15 kJ/kg

Qout = Q41 = mCp(T4 – T1)


qout = Cp(T4 – T1) = 561.48 kJ/kg

Wnet = qin – qout = 875.67 kJ/kg

𝑊𝑛𝑒𝑡
i) 𝜂,Diesel = 𝑞𝑖𝑛
𝜼,Diesel = 0.6093 (60.93%)

ii) Pmax = P2 = P3
𝑘−1
𝑃2 𝑇2 𝑘
= (𝑇 )
𝑃1 1
P2 = 5148kPa (Pmax)

𝑊𝑛𝑒𝑡 𝑊𝑛𝑒𝑡 875.67


iii) MEP = 𝑉 = 1 = 1
𝑚𝑎𝑥 − 𝑉𝑚𝑖𝑛 𝑉1 (1− ) 0.9566(1− )
𝑟 18

MEP= 969.1kPa

REFRIGERATION

Refrigeration Cycle
Heat flows in direction of decreasing temperature, i.e., from high-temperature to
low temperature regions. The transfer of heat from a low-temperature to high-temperature
requires a refrigerator and/or heat pump.
Refrigerators and heat pumps are essentially the same device; they only differ in
their objectives. The performance of refrigerators and heat pumps is expressed in terms of
coefficient of performance (COP):
𝑄𝐿
𝐶𝑂𝑃𝑅 =
𝑊𝑛𝑒𝑡,𝑖𝑛
𝑄𝐻
𝐶𝑂𝑃𝐻𝑃 =
𝑊𝑛𝑒𝑡,𝑖𝑛
Wnet,in = QH - QL
The Reversed Carnot Cycle
Reversing the Carnot cycle does reverse the directions of heat and work
interactions. A refrigerator or heat pump that operates on the reversed Carnot cycle is called
a Carnot refrigerator or a Carnot heat pump.

Fig. 1. T-S diagram and major components for Carnot Refrigerator

The reversed Carnot cycle is the most efficient refrigeration cycle operating
between two specified temperature levels. It sets the highest theoretical COP. The
coefficient of performance for Carnot refrigerators and heat pumps are:

𝑇𝐿
𝐶𝑂𝑃𝑅 =
𝑇𝐻 − 𝑇𝐿
𝑇𝐻
𝐶𝑂𝑃𝐻𝑃 =
𝑇𝐻 − 𝑇𝐿
The Carnot cycle cannot be approximated in an actual cycle, because:
1- executing Carnot cycle requires a compressor that can handle two-phases
2- also process 4-1 involves expansion of two-phase flow in a turbine.

Sample Problem 10.


The food compartment of a refrigerator is maintained at 4°C by removing heat
from it at a rate of 360 kJ/min. If the required power input to the refrigerator is 2 kW,
determine (a) the coefficient of performance of the refrigerator and (b) the rate of heat
rejection to the room that houses the refrigerator.
360
𝑘𝐽/𝑠
𝐶𝑂𝑃𝑅 = 60
2 𝑘𝑊

COPR = 3

Sample Problem 11
A heat engine is used to drive a heat pump. The heat transfers from the heat
engine and the heat pump are rejected to the same sink. The efficiency of the heat engine
is 27%and the COP of the heat pump is 4. Determine the ratio of the total heat rejection
rate to the heat transfer to the heat engine.

𝑊
𝜂=𝑄
1

W = 0.27Q1

𝑄4
COPHP = 𝑊

𝑄4
4= 𝑊

𝑄4
W= 4

0.27Q1 = Q4/4

Q4/Q1 = 1.08

𝑄
𝜂 = 1 - 𝑄2 = 0.27
1

𝑄2
= 0.73
𝑄1

𝑄2 +𝑄4
= 1.08 + 0.73
𝑄1
𝑸𝟐 +𝑸𝟒
= 1.81
𝑸𝟏
The ratio of the total heat rejection rate (Q2+Q4) to the heat transfer to the heat engine
(Q1) is 1.81.
The Ideal Vapor‐Compression Refrigeration Cycle
The vapor-compression refrigeration is the most widely used cycle for refrigerators,
air conditioners, and heat pumps.

Fig. 2. Schematic diagram for ideal vapor-compression refrigeration cycle


Assumptions for ideal vapor-compression cycle:
 irreversibilities within the evaporator, condenser and compressor are ignored
 no frictional pressure drops
 refrigerant flows at constant pressure through the two heat exchangers (evaporator
and condenser)
 heat losses to the surroundings are ignored
 compression process is isentropic

Fig. 3. T-S and P-H diagrams for an ideal vapor-compression refrigeration cycle
1-2: A reversible, adiabatic (isentropic) compression of the refrigerant. The saturated vapor
at state 1 is superheated to state 2.
Wc =h2 − h1
2-3: An internally, reversible, constant pressure heat rejection in which the working
substance is de-superheated and then condensed to a saturated liquid at 3. During this
process, the working substance rejects most of its energy to the condenser cooling water.
qH = h2 − h3
3-4: An irreversible throttling process in which the temperature and pressure decrease at
constant enthalpy. The refrigerant enters the evaporator at state 4 as a low-quality saturated
mixture.
h3 = h4
4-1: An internally, reversible, constant pressure heat interaction in which the refrigerant
(two-phase mixture) is evaporated to a saturated vapor at state point 1. The latent enthalpy
necessary for evaporation is supplied by the refrigerated space surrounding the evaporator.
The amount of heat transferred to the working fluid in the evaporator is called the
refrigeration load.
qL = h1 − h4

Sample Problem 12
In an ideal vapour compression refrigeration cycle, the enthalpy of the refrigerant
at exit from the condenser, compressor and evaporator is 80kJ/kg, 200kJ/kg and 180kJ/kg,
respectively. What is the coefficient of performance of the cycle?

h3 = h4 = 80 kJ/kg
h1 = 180 kJ/kg
h2 = 200 kJ/kg
Wc = h2 – h1 = 200-180 = 20 kJ/kg
Q = h1 – h4 = 180 – 80 = 100 kJ/kg

𝑄 100
COP = 𝑊 =
𝑐 20
COP = 5

Sample Problem 13
In a vapour compression refrigerant plant, the enthalpy values at different points
are:
(i) Enthalpy at exit of the evaporator = 350 kJ/kg
(ii) Enthalpy at exit of the compressor = 375 kJ/kg
(iii) Enthalpy at exit of the condenser = 225 kJ/kg
The refrigerating efficiency of the plant is 0.8. What is the power required per kW of
cooling to be produced?
h3 = h4
Refrigerating effect (Qo) = (h1 – h4) x 𝜂r
= (350 – 225) x 0.8
= 100 kJ/kg
Compressor work (W) = (h2 – h1)
= 375 – 350
= 25 kJ/kg
𝑊 25
The power required per kW of cooling = =
𝑄 100
= 0.25 kW/kW of cooling

Selecting the Right Refrigerant


When designing a refrigeration system, there are several refrigerants from which to
choose. The right choice of refrigerant depends on the situation at hand. The most common
refrigerants are: R-11, R-12, R-22, R-134a, and R-502.

R12: CCl2F2 dichlorofluoromethane, used for refrigeration systems at higher temperature


levels- typically, water chillers and air conditioning (banned due to ozone layer effects)

R22: CHClF2 has less chlorine, a little better for the environment than R12 - used for lower
temperature applications

R134a: CFH2CF3 tetrafluorethane - no chlorine- went into production in 1991-


replacement for R12

Ammonia NH3: corrosive and toxic- used in absorption systems-cheap- high COP

R744: CO2 behaves in the supercritical region- low efficiency

R290: propane combustible.


Many factors need to be considered when choosing a refrigerant:

ozone depletion potential ,global CFC (chlorofluorocarbons) refrigerants


warming potential: allow more ultraviolet radiation into the
earth’s atmosphere by destroying the
protective ozone layer and thus
contributing to the greenhouse effect that
causes the global warming. As a result the
use of some CFCs (e.g. R-11, R-12, and R-
115) are banned by international treaties.
combustibility: all hydro-carbon fuels, such as propane
leak detectability: the refrigerant saturated pressure at the
evaporator should be above Patm
thermal factors: 1- the heat of vaporization of the refrigerant
should be high. The higher hfg, the greater
the refrigerating effect per kg of fluid
circulated.
2- the specific heat of the refrigerant should
be low. The lower the specific heat, the less
heat it will pick up for a given change in
temperature during the throttling or in flow
through the piping, and consequently the
greater the refrigerating effect per kg of
refrigerant
3- the specific volume of the refrigerant
should be low to minimize the work
required per kg of refrigerant circulated
since evaporation and condenser
temperatures are fixed by the temperatures
of the surroundings
4- selection is based on operating pressures
in the evaporator and the condenser.

Other desirable characteristics of refrigerant are: non-flammable, being chemically stable,


and be available at low cost.

Das könnte Ihnen auch gefallen